File Download Area

Information about "Official SAT Practice Test 2013-2014 (2009-2010).pdf"

  • Filesize: 7.84 MB
  • Uploaded: 14/12/2018 20:38:27
  • Status: Active

Free Educational Files Storage. Upload, share and manage your files for free. Upload your spreadsheets, documents, presentations, pdfs, archives and more. Keep them forever on this site, just simply drag and drop your files to begin uploading.

Download Urls

  • File Page Link
    https://www.edufileshare.com/6cf8728f1a732b3b/Official_SAT_Practice_Test_2013-2014_(2009-2010).pdf
  • HTML Code
    <a href="https://www.edufileshare.com/6cf8728f1a732b3b/Official_SAT_Practice_Test_2013-2014_(2009-2010).pdf" target="_blank" title="Download from edufileshare.com">Download Official SAT Practice Test 2013-2014 (2009-2010).pdf from edufileshare.com</a>
  • Forum Code
    [url]https://www.edufileshare.com/6cf8728f1a732b3b/Official_SAT_Practice_Test_2013-2014_(2009-2010).pdf[/url]

[PDF] Official SAT Practice Test 2013-2014 (2009-2010).pdf | Plain Text

InsIde : About the SAT ® Test-taking advice and tips An of\bcial practice test Find more free and affordable practice tools at sat.org/practice . effective september \b013 A little practice goes a long way. Getting Ready for the SAT ®

Do your best on test day. Practice with real SAT tests and get helpful tips. sat.org/practice n Free SAT Practice Tools n The Official SAT Study Guide \b (also available with DVD!) n The Official SAT Online Course \b © 2013 The \bollege Board. Get more Free An D AFF \brDA ble SAT ® practice tools from the test maker

Getting Ready for the SAT 1 © 2013 The College Board. College Board, achieve more, \bdvanced Placement, \bdvanced Placement Program, \bP, S\bT, Student Search Service and the acorn logo are registered trademarks of the College Board. My College QuickStart, S\bT Subject Tests, Skills Insigh\,t, The Official S\bT Study Guide, The Official S\bT Question of the D\,ay, The Official S\bT Online Course, The Official S\bT Subject Tests in Mathematics\, Levels 1 & 2 Study Gu\,ide, The Official S\bT Subject Tests in U.S. and W\,orld History Study Guide and The Official Study Guide for all S\bT Subject Tests are trademarks owned by the College Board. PS\bT/NMSQT is a registered trademark of the College Board and National Merit Scholarship Corporation. \bll other pr\,oducts and services may be trademarks of their respective owners. Visit the College Board on the Web: www.collegeboard.org. Introduction to the SAT ® and the College Board If you plan to register for the S\bT, you can use this boo\,k to get familiar with the test. Remember, if you have access to the Internet you can find everything in this b\,ooklet and more at sat.org/practice. Contents Introduction to the S\bT and the College Board ..................\,.................1 The Critical Reading Section ..................\,..................\,..................\,............. 5 The Mathematics Secti\,on ..................\,..................\,..................\,................. 11 The Writing Section ..................\,..................\,..................\,..................\,......... 25 Official S\bT Practice Test ..................\,..................\,..................\,................. 33 Scoring the Essay ..................\,..................\,..................\,..................\,.............. 81 Scoring the Official S\bT Practice Test ..................\,..................\,.............82 Student Search Service® Student Search Service® is a free program that connects you with colleges and universities across the country that are looking for students just like you. If you take the PS\bT/NMSQT®, the S\bT, S\bT Subject Tests™ or any \bP® Exam, you can be included \,in this free service. Here’s how it works: During S\bT or S\bT Subject Test registration, indicate that you want to be a part of Student Search Service. Your name is made available for access, along with other\, information such as your address, high school grade point average, date of birth, grade level, high school, ema\,il address, intended college major and extracurricular activiti\,es. Colleges and scholarship programs then use Search to help them locate and recruit students wit\,h characteristics that they find to be a good match with their programs. This is a great way for you to get information about colleges with which you may not be familiar. Here are some points to keep in mind about S\,tudent Search Service: ■ Being part of Student Search Service is voluntary. While most students who take the PS\bT/NMSQT, \bP Exams, S\bT or S\bT Subject Tests participate in this service, you may take the test even if you don’t take part in Student Search Service. ■ Colleges participating in Stude\,nt Search Service never receive student scores or phone numbers. Colleges can ask for names of students within \,certain score ranges, but your exact score is not reported. ■ Being contacted by a college doesn’t mean you have been admitted. The colleges and organizations that participate want to find students who\, fit in with their \,environment, classes, programs, scholarships and special a\,ctivities. However, you can be considered for admission only after you apply. Student Search Service is simply a way for colleges to reach prospective students like you and inform them of their opportunities. ■ Student Search Service will share your contact information only with colleges and qualified no\,nprofit educational or \, scholarship programs. Your name will never be sold to a commercial marketing firm or retailer of merchandise or services (such as test prep). Wh\b Should You Take the SAT? The S\bT is an essential passport for your college admission journey. The S\bT gives colleges what they want: a showcase of your skills and potential. The College Board is here to help guide you throughout the college-going process so that you can find the right college fit and best path forward to future opportunities. It’s more than just a test. The S\bT can also help connect you to our college planning resources. Go to bigfuture.org to start your journey. Learn how other students like you made their college choices. Find out more about the application process, access financial aid tools, explore majors and more — all to help you narrow down your choices and find the right college. It’s one path to financial support and scholarships. The S\bT can help connect you to scholarship organizations, and many colleges use it for merit awards in addition to admission. Take a look at the “Student Search Service” information on this page to learn more. It’s fair to ever\bone. The S\bT was first created to help level the playing field in college admission. It still does so today. The S\bT is the most rigorously researched and designed test in the world, which ensures that students from all backgrounds have an equal chance to succeed.

2 Getting Ready for the SAT What Does the SAT Measure? The S\bT measures the skills you have learned in and o\,utside of the classroom and how well you can apply that k\,nowledge. It tests how you think, solve problems and communicate. The test is composed of three sections: ■ Critical Reading, which includes sen\,tence completion and passage-based reading questions. ■ Mathematics, which includes mult\,iple-choice and student- produced response questions \,based on the math t\,hat college- bound students typically learn duri\,ng their first three years of high school. ■ Writing, which has multiple\,-choice questions and a \,written essay. You have three hours and 45 minutes of testing time, plus t\,hree five-minute breaks, for a total of four hours to complete the entire test. \bll multiple-choice questions are scored the same way: one point for each correct answer, and one-quarter point subtracted for a wrong answer. No points are subtracted for answers left blank or for incorrect student-produced response questions \, (see page 22). The total score for each section is \,on a 200- to 800-point scale, wi\,th 10-point intervals. How Can You Get Read\b for the Test? Get Started Now ■ Take challenging classes, study hard, and read and write outside of the classroom. ■ Take the PS\bT/NMSQT in your sophomore and/or junior year. Once you get your results, sign in to My College QuickStart™ to get a personalized planning kit to help you start getting ready for the S\bT. ■ Review the sample questi\,ons, test-taking approaches and directions in this boo\,klet. ■ Take advantage of our low-cost practice materials, with the lessons, practice questions, and pr\,actice exams in The Official S\bT Online Course™, and The Official SAT Study Guide™ Third Edition (also available with DVD). Use Our Online Resources \bt sat.org /practice you’ll find a wealth of practice tools to help you put your best foot forward on test day. ■ Create your own study plan and \,practice with The Official S\bT Question of the Day™, also available via Twitter or our mobile app. ■ Take an official S\bT practice test online so you can get it scored automatically and review the answer explanations. You can also take the practice test included in thi\,s booklet. ■ With S\bT Skills Insight™ \,you can see what you need to get to the next level on the S\bT. Approaches to Taking the SAT Know What to Expect Use the information in this bo\,oklet and at sat.org /practice to help you be ready on test day. ■ Become familiar with the test\b Study the descript\,ions of the S\bT sections, so you’re not surprised or\, confused on test day. ■ Learn the test directions\b The directions for answering the questions in this \,booklet are the same as those\, on the actual test. If you become familiar with the di\,rections now, you’ll leave yourself more time to answer the questions w\,hen you take the test. ■ Review the sample questions\b The more familiar you are with the question formats, the more comfortable you’ll feel when you see similar ques\,tions on the actual\, test. In particular, be sure to practice writing answers to the student-produced response mathematics \,questions (see page 22). ■ Understand how the tests are scored\b You get one point for each right answer and lose a fraction of a point for each wrong answer. You neither gain nor lose points for omitting an answer. Hard questions count the same amount as easier questions. Use These Test-Taking Strategies ■ Listen carefully to instructions and follow the directions given to you\b Be sure to fill in the Form Code, Test Form and Test Book Serial Nu\,mber fields as directed on page 1 of the answer sheet. If they are not correct, we won’t be able to score your test. ■ Read carefully\b Consider all the cho\,ices in each question\,. \bvoid careless mistakes that will cause \,you to lose points. ■ Answer the easy questions first\b Work on less time- consuming questions \,before moving on to the more difficult ones. Questions in\, each section are generally ordered from easiest to hardest. The exception is in the p\,assage-based reading questions, w\,hich are ordered according to the logic and organization of each passage. ■ Eliminate answer choices that you know are wrong\b Cross them out in your test booklet so tha\,t you can clearly see \,which choices are left. ■ Make educated guesses or skip the qu\Westion\b If you have eliminated the choices that you know are wrong, guessing is your best strategy. However, if you cannot eliminate any of the answer choices, it is best to skip the questio\,n. You will lose points for incorrect answers. No points are deducted if an answer is left blank. ■ Use your test booklet as scratch paper\b Use it to make notes or write down ideas. What you write in the booklet w\,ill not affect your score. ■ Circle the questions you’ve skipped\b This will help you keep track of which questio\,ns you’ve skipped. ■ Check your answer sheet regularly\b Make sure you are in the right place. Check the number of t\,he question and th\,e number on the answer sheet every few questions. This is especially important when you skip a question\,. Losing your place on the answer sheet can cost you time and even points.

Getting Ready for the SAT 3 ■ Keep your answer sheet neat\b The answer sheet is scored by a machine, which ca\,n’t tell the difference between an answer and a doodle. I\,f the machine reads what appear t\,o be two answers for one question, it\, will consider the questi\,on unanswered. ■ Work at an even, steady pace and limit your time on any one question\b \bll questions are worth the same number \,of points. If you need a lot of ti\,me to answer a question, go on to the next one. Later, you may have time to return to the question you skipped. Your goal is to spend time on the\, questions that you are most likely to answer correctly. ■ Keep track of time\b Occasionally check your progress so that you know where you are and how much time is left. ■ Always use a No\b 2 pencil\b \bll answer sheet circles must be filled in darkly an\,d completely with a No. 2 pencil. If you need to erase an answer, erase it as completely as possible. The S\bT essay must be written with a No. 2 pencil. Essays written in pen will receive a score of zero. On Test Da\b Make sure that you read and understand our policies \,and requirements for taking the S\bT. These are available in two places: ■ Online at sat.org /test-da\b ■ In the Registration Guide to the SAT and SAT Su\bject Tests You will need to bring the following to the test center: ■ Your \bdmission Ticket, which is required for entry to the test center. ■ \bn acceptable photo ID. Be sure to check online or i\,n the registration guide about \,what ID you can use. ■ Two No. 2 pencils and a s\,oft eraser. ■ \bn approved calculator with fresh batteries. For more information on calculat\,ors, see pages 11–12. — Make sure your calculator is in good working order. — You may bring additional\, batteries and a backup calculator to the test center. Test center staff will not h\,ave batteries or calculators for your use. — You may not share a calculator with another test-taker\b Test Scores Scores are available for free at sat.org /scores several weeks after each test is given. You can also get your scores, for a fee, by telephone. Call Customer Service at 866-756-7346 in the United States. From outside the Unit\,ed States, dial +1-212-713-7789. How the Test Is Scored The first step in scoring is to calculate the raw score for each section. The second step is to convert that raw score to a scale of 200 to 800. The raw scores are calculated as follows: ■ For each correct answer, add (+) 1 point. \, ■ For each incorrect multiple-choice answer, subtract (–) 1/4 point. ■ For each incorrect student-produced response, no points \,are subtracted. ■ For each question no\,t answered, no points are subtracted. SAT Writing Subscores In addition to an overall writing score, you receive two writing subscores — one for your essay, and one for the multiple-cho\,ice questions. ■ The essay subscore is a raw score ranging from 2 to 12. The essay raw score contributes about 30 percent toward your total writing score. (Note: Essays that are not written on the essay assignment, not writt\,en in No. 2 pencil, or not \, considered legible after several attempts at reading will receive a zero.) ■ The multiple-choice writing subscore on your score report has been converted from a raw score to a scale of 20 to 80. The multiple-choice raw score contributes about 70 percent to your overall writing score. M\b SAT Online Score Report My S\bT Online Score Report gives you personalized, practical information about your S\bT results, including h\,ow your score compares to those of the total group of test-takers and how you performed on each sectio\,n of the test. It also allows you to access and print your essay to share with your teachers or counselor. Link to your report at collegeboard.org /satscores. Score Choice ™ Score Choice ™ gives you the option to choose which scores to send colleges based on what e\,ach college wants to receive. You can select which scores to send each college by test date for the S\bT and by individual test taken for S\bT Subject Tests™. You can choose scores from one, several or all S\bT test dates in your record. Score Choice is optional, so i\,f you don’t actively choose to use it, all of your scores will be sent au\,tomatically with your score report. Since most colleges only consider your best scores, you should still feel comfortable reporting scores from all of your tests. About the College Board The College Board is a mission-driven not-for-profit organization that connects students to college success and opportunity. Founded in 1900, the College Board was created to expand access to higher education.\, Today, the membership association is made up of over 6,000 of the world’s leading education\,al institutions and i\,s dedicated to promoting excellence and equity in education. Eac\,h year, the College Board helps more than seven million students\, prepare for a successful transition to college through programs and services in college readiness and college success — including the S\,\bT® and the \bdvanced Placement Program®. The organization also serves the education community through research and advocacy on behalf of students, educators and schools. For further information, visit www.collegeboard.org.

4 Getting Ready for the SAT Protecting Your Privac\b Telemarketing and Internet Scams From time to time, we receive reports of phone scams in\, which callers posing as employees of the College Board contact students and families attempting to sell test preparation products or otherwise\, requesting sensitive personally identifying\, information, such as cr\,edit card and social securi\,ty numbers. Some of these caller\,s engage in illegal spoofing to make it seem as if the call is coming from the actual company. These calls do not come from the College Board. The College Board does not make unsolicited phone calls to students or families requesting this t\bpe of information. This type of activity, known as telemarketing fraud, is a crime. Sho\,uld you receive an unsolicited phone call from someone claiming \,to work for the College Board, including where your Caller ID indi\,cates that the telephone number originates from a College Board location, do not \,provide the caller with any personal information. Representatives of the College Board only make calls or send text messages to students and the\,ir families in response to student- generated inquiries and/or to provide or gather information about a test or program for which the studen\,t registered or regarding preparation for college and the applicati\,on process. Should you have a question about\, the origin of a ph\,one call you have received in which the caller\, claims to be from the College Board, contact Customer Service. Safet\b and Securit\b Tips 1. Be wary of unsolicited contacts, whether via\, telephone or email. 2. Remember that the College Board will never contact you to ask you to send your credit card, bank account, or password information over the telephone or through email. 3. Never supply credit card information to someone who calls\, or emails you. 4. If you suspect you have received a fraudulent call or em\,ail, contact the Federal Trade Commission (FTC) and your local authorities and pr\,ovide them with all \,the details. 5. Keep in mind that if \,an offer appears too good to be true, it probably is. 6. To make a complaint and to obtain more information about protecting yourself from telephone and Internet scams, visit the FTC's Consumer Information site at www.ftc.gov/bcp/ menus/consumer/phone.shtm.

Getting Ready for the SAT 5 The Critical Reading Section The critical reading section gives you a chance to show how well you understand what you read. This section has two types of questions: ■ Sentence completions (19 ques\,tions) ■ Passage-based reading (48 questions) Note: Calculators may not be on your desk or used o\Wn the critical reading section of t\Whe SAT\b Approaches to the Critical Reading Section ■ Work on sentence completion questions\, first. They take less time to answer than the passage-based reading questions. ■ The difficulty of sentence completion questions\, increases as you move through the section. ■ Passage-based reading questions d\,o not increase in difficulty from easy to hard. Instead, they follow the logic of the passage. ■ The information you need to answer each reading question is always in the passage(s). Reading carefully is the key to finding the correct answer. Don’t be misled b\,y an answer that looks correct but is not supp\,orted by the actual text of the passage(s). ■ Passage-based reading questions o\,ften include line num\,bers to help direct you to the relevant part(s) of the passage. If one word or more is quoted exactly from the passage, the line number(s) where that quotation ca\,n be found will appear in the test question. You may have to look elsewhere in the passage, however, in order to find support for the best answer to the question. ■ Do not jump from passage to passage. Stay with a passage until you have answered as many questions as you can before you proceed to the next passage. ■ In your test booklet, mark e\,ach question you skip so that you can easily go back to it later if you have time. ■ Remember that all qu\,estions are worth one point regardless of the type or difficulty. Sentence Completions Sentence completion questions\, measure your ■ knowledge of the m eanings of words; and ■ ability to understand how the different parts of a sentence fit together logically. \birections Each sentence below has one or two blanks, each blank indicating that something has been omitted. Beneath the sentence are fi\be words or sets of words labeled A through E. Choose the word or set of words that, when inserted in the sentence, best fits the meaning of the sentence as a whole. Example: Hoping to ------- the dispute, negotiators proposed a compromise that they felt would be ------- to both labor and management. (A) enforce . . useful (B) end . . di\bisi\be (C) o\bercome . . unattracti\be (D) extend . . satisfactory (E) resol\be . . acceptable a b c d , Answering Sentence Completion Questions One way to answer a sentence completion question \,with two missing words is to focus first on just one of t\,he two blanks. If one of the words in an answer choice is logically wrong, then you can eliminate the entire choice from consideration. ■ Look at the first blank in the ab\,ove example. Would it make sense to say that “negotiators” who have “proposed a compromise” were hoping to enforce or extend the “dispute”? No, so neither (\b) nor (D) can be the correct answer. ■ Now you can focus on the second blank. Would the “negotiators” have proposed a compromise that they believed would be divisive or unattractive to “both labor and \, management”? No, so (B) and (C) can be eliminated, and only choice (E) remains. ■ \blways check your answer by reading the entire sentence with your choice filled in. Does i\,t make sense to say, “Hoping to resolve the dispute, negotiators proposed a compromise that they felt would be acceptable to both labor and management”? Yes. Correct answer: (E) / Difficulty level: Easy Sample Questions 1. Because King Philip\,’s desire to make Spain the dominan\,t power in sixteenth-century Europe ran counter to Queen Elizabeth’s insistence on autonomy for England, ------- was -------. (\b) reconciliation . . assured (B) warfare . . avoidable (C) ruination . . i\,mpossible (D) conflict . . inevitable (E) diplomacy . . s\,imple

6 Getting Ready for the SAT Be sure to look for key words and phrases as you read each sentence. Words such as although, however, if, \but and since are important to notice because they signal how the different parts of a sentence are logically related to each other. Words such as not and never are important because they indicate negation. In the example above, the entire sentence hinges on a few key words: “Because something ran counter to something else, \blank was \blank.” ■ The word “because” indicat\,es that the information in the first part of the sentence (the part before the comma) explains the reason for the situation d\,escribed in the sec\,ond part. The first part states that what King \,Philip wanted (domination for Spain) “ran counter to” what Queen Elizabeth wanted (independence for England). ■ Given that there was such a fundamenta\,l disagreement between the two monarchs, would reconciliation be assured? Unlikely. ■ Would warfare be avoidable? Hardly; warfare might be unavoidable. ■ Would ruination be impossible? No. ■ Would diplomacy be simple? Not necessarily. ■ Only choice (D) fits logically wit\,h the key words in the sentence: Because what one person wanted ran counter to what another person wanted, conflict was inevitable. Correct answer: (D) / Difficulty level: Medium 2. There is no doubt that\, Larry is a genuine -------: he\, excels at telling stories that fascinate his listeners. (\b) braggart (B) dilettante (C) pilferer (D) prevaricator (E) raconteur Some sentence completion questions\, contain a colon. This is a signal that the \,words after the colon define or directly clarify what came before. In this case, “h\,e excels at telling stories that fascinate his listeners” serves to define the word raconteur, choice (E). None of the \,other words is directly defined by this clause. ■ \b braggart may or may not excel at telling stories and may actually annoy listeners. ■ \b dilettante is someone who dabbles at a career or hobby and so may not excel at anything. ■ \b pilferer steals repeatedly, in small quantiti\,es; this has nothing to do with storytelling. ■ \b prevaricator tells lies, but not \,necessarily in an accomplished or fascinating way; and the sentence refers to stories, not lies. You should choose th\,e word that best fits th\,e meaning of the sentence as a whole, and o\,nly choice (E) does so. Correct answer: (E) / Difficulty level: Hard Passage-Based Reading The passage-based reading questions o\,n the S\bT measure your ability to read and think carefully about several different passages ranging in length from about 100 to about 850 words. Passages are taken from a variety of fields, includin\,g the humanities, soci\,al studies, natural sciences and fiction or literary nonfiction. They vary in style and can include\, narrative, argumentative and expository elements. Some \,selections consist of a pair of related passages on a shared issue or theme; in so\,me of the questions, you are asked to compare and contrast these passages. The following kinds of quest\,ions may be asked about a passage: ■ Vocabulary in Context: These questions ask\, you to determine the meanings of words from their context in the reading passage. ■ Literal Comprehension: These questions assess your understanding of significa\,nt information directly stated in the passage. ■ Extended Reasoning: These questions meas\,ure your ability to synthesize and analyze information as well as to evaluate the assumptions made and \,the techniques used by the author. Most of the reading questions fall into this category. You may be asked to identify cause and\, effect, make inferences, recognize a main idea or an\, author’s tone, or follow the logic of an analogy or an\, argument. Answering Passage-Based Reading Questions Following are samples of the kin\,ds of reading passages and questions that may appear on your test. For each set of sampl\,e materials: ■ Read the passage carefully. ■ Decide on the best \,answer to each question. ■ Read the explanation for the correct answer. Some of the reading passages on the S\bT are as short as a paragraph or two, about 100 words in length. You will also find one or more pairs of related short passages in each edition \,of the test. Such material will be followed by one to five questions that measure the same kinds of \,reading skills that\, are measured by the questions following longer passages.

Getting Ready for the SAT 7 Line 5 10 15 \birections The passages below are followed by questions based on their content; questions following a pair of related passages may also be based on the relationship between the paired passages. Answer the questions on the basis of what is stated or implied in the passages and in any introductory material that may be pro\bided. Sample Questions Questions 3-4 are based on the following passage\b “The rock was still wet. The animal was glistening, like it was still swimming,” recalls Hou Xianguang. Hou \biscovere\b the unusual fossil while surveying rocks as a paleontology gra\buate stu\bent in 1984, near the Chinese town of Chengjiang. “My teach- ers always talke\b about the Burgess Shale animals. It looke\b like one of them. My han\bs began to shake.” Hou ha\b in\bee\b foun\b a Naraoia like those from Cana\ba. However, Hou’s animal was 15 million years ol\ber than its Cana\bian relatives. Some questions ask you to recognize the meaning of a w\gord as it is used in the passage. 3. In line 4, “surveying” most nearly m\,eans (\b) calculating the value of (B) examining comprehensively (C) determining the bounda\,ries of (D) polling randomly (E) conducting a statist\,ical study of The word “surveying” has a number \,of meanings, several of which are included in the c\,hoices above. In the context of this passage, however, only (B) makes sense. \b student\, in the field of “paleontology” is one who \,studies prehistoric life as recorded in fossil remains. One of the a\,ctivities of a paleo\,ntology student would be to examine rocks carefully and “comprehensively” while looking for fossils. ■ (\b), (C) and (E) are incorrect because someone \,who studies fossils would not calculate the “value” of rocks, or determine the “boundaries” o\,f rocks, or conduct a “statistic\,al study” of rocks. ■ (D) is wrong because “pollin\,g” rocks makes no sense at all\,. Correct answer: (B) / Difficulty level: Easy You may \be asked to make an inference or draw a conclusion a\bout a statement made in the p\gassage. 4. It can be inferred that Hou Xiangu\,ang’s “hands began to shake” (line 9) because\, Hou was (\b) afraid that he might l\,ose the fossil (B) worried about the i\,mplications of his fi\,nding (C) concerned that he might\, not get credit for his work (D) uncertain about the aut\,henticity of the fossil (E) excited about the magnit\,ude of his discovery In the passage, Hou states that the fossil that he found “looked like” certain other fossils that his “teachers always talked about.” He understands almost immedi\,ately, therefore, the significance of what he has found, and so (E) is\, the correct answer: Hou’s hands were shaking because h\,e was “excited about the magnitude of his dis\,covery.” ■ (\b) is wrong because there is no suggestion that Hou was “afraid that he might l\,ose the fossil.” ■ (B) and (C) are wrong because the pas\,sage does not indicate that Hou was “worried about” his \,discovery or “concerned that he might not g\,et credit.” The passage indicates only that Hou recognized that he had found something valuable. ■ (D) is wrong because Hou’s immediate reaction is that he thinks he has found an important fossil. The first two sentences of the passage dramatize the discovery; it is Hou’s excitement, not his uncertainty, that causes him to tremble. Correct answer: (E) / Difficulty level: Easy Questions 5-8 are based on the following passages\b These two passages were adapted from auto\biographical works. In the first, a playwright descri\bes hi\gs first visit to a theater in the 1930s; in the second, an eighteenth-century writer descri\bes two visits to theaters in London. Passage 1 I experience\b a shock when I saw a cur- tain go up for the first time. My mother ha\b taken me to see a play at the Schubert Theater on Lenox Avenue in Harlem in New York City. Here were living people talking to one another insi\be a large ship whose \beck actually heave\b up an\b \bown with the swells of the sea. By this time I ha\b been going to the movies every Satur\bay afternoon —Charlie Chaplin’s little come\bies, a\bven- ture serials, Westerns. Yet once you knew how they worke\b, movies, unlike the stage, left the min\b’s grasp of reality intact since the happenings were not in the theater where you sat. But to see the \beck of the ship in the theater moving up an\b \bown, an\b people appearing at the top of a la\b\ber or \bisappearing through a \boor—where \bi\b Line 5 10

8 Getting Ready for the SAT an\b, incapable of the anticipation, I repose\b my shut eyes in a sort of resignation upon my mother’s lap. It rang the secon\b time. The curtain \brew up—an\b the play was Artaxerxes ! Here was the court of ancient Persia. I took no proper interest in the action going on, for I un\berstoo\b not its import. Instea\b, all my feeling was absorbe\b in vision. Gorgeous costumes, gar\bens, palaces, princesses, passe\b before me. It was all enchantment an\b a \bream. After the intervention of six or seven years I again entere\b the \boors of a theater. That ol\b Artaxerxes evening ha\b never \bone ringing in my fancy. I expecte\b the same feelings to come again with the same occa- sion. But we \biffer from ourselves less at sixty an\b sixteen, than the latter \boes from six. In that interval what ha\b I not lost! At six I knew nothing, un\berstoo\b nothing, \biscriminate\b nothing. I felt all, love\b all, won\bere\b all. I coul\b not tell how, but I ha\b left the temple a \bevotee, an\b was returne\b a rationalist. The same things were there materially; but the emblem, the reference, was gone. The green curtain was no longer a veil, \brawn between two worl\bs, the unfol\b- ing of which was to bring back past ages, but a certain quantity of green material, which was to separate the au\bience for a given time from certain of their fellows who were to come forwar\b an\b preten\b those parts. The lights—the orchestra lights—came up a clumsy machinery. The first ring, an\b the secon\b ring, was now but a trick of the prompter’s bell. The actors were men an\b women painte\b. I thought the fault was in them; but it was in myself, an\b the alteration which those many centuries—those six short years—ha\b wrought in me. * Pilasters are ornamental columns set into walls. they come from an\b where \bi\b they go? Obviously into an\b out of the real worl\b of Lenox Avenue. This was alarming. An\b so I learne\b that there were two kin\bs of reality, but that the stage was far more real. As the play’s melo\bramatic story \bevelope\b, I began to feel anxious, for there was a villain on boar\b who ha\b a bomb an\b inten\be\b to blow everybo\by up. All over the stage people were looking for him but he appeare\b, furtive an\b silent, only when the searchers were facing the other way. They looke\b for him behin\b posts an\b boxes an\b on top of beams, even after the au\bience ha\b seen him jump into a barrel an\b pull the li\b over him. People were yelling, “He’s in the barrel,” but the passengers were \beaf. What anguish! The bomb woul\b go off any minute, an\b I kept clawing at my mother’s arm, at the same time glancing at the the- ater’s walls to make sure that the whole thing was not really real. The villain was finally caught, an\b we happily walke\b out onto sunny Lenox Avenue, save\b again. Passage 2 I was six years ol\b when I saw my first play at the Ol\b Drury. Upon entering the theater, the first thing I behel\b was the green curtain that veile\b a heaven to my imagina- tion. What breathless anticipations I en\bure\b! I ha\b seen something like it in an e\bition of Shakespeare, an illustration of the tent scene with Diome\be in Troilus and Cressida . (A sight of that image can always bring back in a measure the feeling of that evening.) The balconies at that time, full of well-\bresse\b men an\b women, projecte\b over the orchestra pit; an\b the pilasters* reaching \bown were a\borne\b with a glister- ing substance resembling sugar can\by. The orchestra lights at length rose. Once the bell soun\be\b. It was to ring out yet once again— 60 65 70 75 80 85 \b0 \b5 20 25 30 35 40 45 50 55

Getting Ready for the SAT 9 Following are four sample questions about this pair of related passages. In the test, as many as 13 questions may appear with a passage of this length. Some questions will focus on Passage 1, others will focus on Passage 2, and about half of the questions following each pair of passages will focus on the relationships between the passages. Some questions require you to identify shared ideas or similari\gties \between the two related passages. 5. The authors of both passages describe (\b) a young person’s sense of wonder at first seeing a play (B) a young person’s desire to become a playwright (C) the similarities between plays and other art forms (D) how one’s perception of the theat\,er may develop over time (E) the experience of reading a play and then seeing \,it performed To answer this question, y\,ou have to figure out what these t\,wo passages have in common. The subject of Passage 1 is a child’s first visit to see a play performed in a theater, and how captivated he was by the entire experience. Passage 2 describes two different visits to the theater; at age six the child is \,entranced by the spectacle of t\,he performance, but “after the intervention of six or seven years” the older and n\,ow more knowledgeable child is not so imp\,ressed. (\b) is the correct answer because all of Passage 1 and the first half of Passage 2 describe “a young person’s sense of wonder at first seeing a play.” ■ (B) is wrong; even though the intr\,oduction to these passages reveals that one of th\,e authors is a “playwright,” there is no mention in ei\,ther passage of a “desire to become a playwright.” ■ (C) is wrong because Passage 1 mentions differences rather than “similarities”\, between plays and movies, and Passage 2 does not mention an\,y “other art forms” at all. ■ (D) is wrong because only Passage 2 discusses “how one’s perception of the theat\,er may develop over time”—this subject is unmentio\,ned in Passage 1. ■ (E) is wrong because there is no reference in either passage to “the experience of reading a play.” Correct answer: (A) / Difficulty level: Easy Some questions assess your comprehension of information that is directly stated in a passage. 6. The “happenings” men\,tioned in line 14 \,refer to the (\b) work undertaken to produce a movie (B) events occurring in the street outside the theater (C) fantasies imagined b\,y a child (D) activity captured on the movie screen (E) story unfolding on the stag\,e To answer this question correctly, you have to understand lines 11–15, a rather complex sentence that makes an important distinction in Passage 1. The author indicates that, unlike plays, movies leave “the mind’s grasp of reality intact,” because the “happenings” in a \,movie are not occurring in the actu\,al theater. Instead, images are projected on a screen in the theater. Thus (D) is the correct answer; the word “happenings” refers to the “activity captured on the movie screen.” ■ (\b) and (B) are wrong because, when y\,ou insert them in place of the word “happenings,” the sentence in lines 11–15 makes no sense. ■ (C) is wrong; even if the movies being referred to include “fantasies” in them, \,they are not “imagined by a child” but are actually projected on the movie screen. ■ (E) is wrong because, in lin\,e 14, “happenings” \,refers to the “story unfolding” in a movie, not “on the stage.” Correct answer: (D) / Difficulty level: Medium

10 Getting Ready for the SAT You may \be asked to recognize the author’s tone or attitude in a\g particular part of a passage, or in the passage as a whole. 7. In the final sentence of Passage 2 (“I thought . . . in\, me”), the author expresses (\b) exultation (B) vindication (C) pleasure (D) regret (E) guilt Even though this que\,stion focuses on a single \,sentence, you must understand the context in which the sta\,tement occurs in order to determine the feeling expressed by the author. In the second paragraph of Passage 2, the author sta\,tes that the experience of attending a play at age 12 or 13 was much different than at age 6. “The same things were there materially” in the theater, but the older chi\,ld knew much more than the younger one about what \,was going on. Ironically, this increased knowledge actually decreased the author’s pleasure in attending the play. “In that interval what had I not \,lost!” the author \, exclaims in line 78. Where the younger child saw nobles in “the court of ancient Persia,” the older child s\,aw “men and women painted.” Thus the final sentence of Passage 2 expresses “regret” concerning the changes that “those many centuries — those six short years — had wrought” in the auth\,or. (D) is the correct answer. ■ (\b) and (C) are incorrect because the aut\,hor does not feel “exultation” about o\,r take “pleasure” in the “alteration” that has occurred; on the contrary, the author lament\,s it. ■ (B) is incorrect because there is no expression of “vindication” in th\,e final sentence; the author is n\,ot trying to justify, support, or defend the experiences described in the\, passage, but rather to explain the changes that have occurred due to the passing of time. ■ (E) is incorrect because, even though the final\, sentence states that the “fault” was not in the actors but in the now more knowledgeable child, the au\,thor feels no “guilt” about the change. There is no way to avoid the passage of time (and the learning \,that goes along with it)\,. \bging is not the \, child’s “fault,” but the loss of a youthful sense of wonder and innocence can still cause r\,egret. Correct answer: (D) / Difficulty level: Hard Some questions require you to determine and compare the primary purpose or main idea\g expressed in each passage. 8. Which of the following best describes\, the difference between Passages 1 and 2 ? (\b) Passage 1 remembers an event with fondness, while Passage 2 recalls a similar event with bitter detachment. (B) Passage 1 considers why the author responded to the visit as he did, wh\,ile Passage 2 supplies the a\,uthor’s reactions without fur\,ther analysis. (C) Passage 1 relates a story from a number of different perspectives, while Passage 2 maintains a sin\,gle point of view. (D) Passage 1 treats the visit to the theater as a disturbing\, episode in the aut\,hor’s life, while Passage 2 describes the author’s visit as joyful. (E) Passage 1 recounts a childhood experience, while Passage 2 examines how a similar experience changed over time. This question asks you to do two things: first, understand the overall subject or purp\,ose of each passage; second, recognize an important “difference between” the two. The correct answer is (E) because the entire first passage does indeed tell the story of a particular “childhood experience” — a trip to the theater — whereas the second passage describes two different trips to the theater and how the “experience changed over time.” ■ (\b) is wrong because there is neither bitterness nor “detachment” in Passage 2. In fact, the first paragraph of Passage 2 expresses excitement and “enchantment,” and the second paragraph expresses disappointment \,and regret. ■ (B) is wrong because Passage 2 includes a great deal more than just “the author’s reactions” to visiting the thea\,ter; most of the second paragraph provides “further analysis” of what had changed and why the reactions to the two visits were so different. ■ (C) is wrong because it reverses the two narrative approaches in this pair\, of passages. Passage 1 “maintains a single point of vi\,ew,” that of the youthful first-time theatergoer, whereas the author of P\,assage 2 presents at least two “different perspectives,” that of the enchan\,ted six-year-old and of the old\,er child returning to the theater. ■ (D) is wrong because the aut\,hor of Passage 1 does not find his first visit to the theater “disturbing” in a n\,egative way. \blthough he feels “shock” when t\,he curtain goes up and anxiety during the play, these responses merely indicate how effective and “real” the performance was for him. In the end, the child and his mother walked “happily” out of the theater. Correct answer: (E) / Difficulty level: Easy

Getting Ready for the SAT 11 The Mathe\batics Section The mathematics secti\,on of the S\bT contains two types of questions: ■ Standard multiple-choice (44 questions) ■ Student-produced response questions \,that provide no answer choices (10 questions) Some questions are like those you may have seen in your mathematics courses. The ability to reason logically in \,a variety of situations, some \,of which may be new to you, is tested throughout. Approaches to the Mathematics Se\hction ■ Familiarize yourself with the directions ahead of ti\Wme\b ■ The test does not require you to memorize formulas\b Commonly used formulas are provided in the test booklet at the beginning of ea\,ch mathematics sectio\,n. It is up to you to decide which formula is appropriate to a question. ■ Read the problem carefully\b Look for key words that tell you what the problem is asking. \bs\,k yourself the following questions before you solve each problem: What is the \, question asking? What do I know? ■ With some problems, it may be useful to draw a sketch or diagram of the given information\b ■ Use the test booklet for scratch work\b You are not expected to do all the reasoning and figuri\,ng in your head. You will not receive credit for anything written in the booklet, \,but you will be able to check your work easily later. ■ Decide when to use a calculator\b (See Calculator Tips.) ■ Eliminate choices\b If you don’t know the correct answer to a question, try som\,e of the choices. It’s sometimes easier \, to find the wrong answers than the correct one. On some questions, you can eliminate all the incorrect choices. ■ Make sure your answer is a reasonable answer to the question asked\b This is especially t\,rue for student-produced response questions, \,where no answer choices are given. ■ All figures are drawn to scale unless otherwise indicated\b Calculator Polic\b You will not be all\,owed to share calculators. You will be dismissed and your scores will be canceled if you use your calculator to share information during the\, test or to remove test questions or answers from the test room. Calculator Tips ■ Remember to bring your calculator on test day\b Calculators will not be available at the test center. You should be familiar with how to use the calculator you bring to the test. ■ Make sure your calculator is in good working order and that its batteries are fresh\b If your calculator fails during testing and you have no backup, you will have to complete the test without it (or cancel your scores for the entire test). ■ Don’t buy an expensive, sophisticated calculator just to take the test\b \blthough you can use them for the test, more sophisticated calculators are not required for any problem. ■ Don’t try to use a calculator on every question\b First, decide how you will solve the problem, and then deci\,de whether to use the calculator. The calculator is meant to aid you in problem solving, not t\,o get in the way. ■ Get your thoughts down before using your calculator\b It may help to do scratch work in the test booklet. ■ Take the practice test with a calculator at hand\b This will help you determine which types of questions y\,ou should use your calculator to answer. Acceptable Calculators We recommend that you bring a calculat\,or to use on the mathematics section \,of the S\bT. Every question on th\,e test can be solved without a calcul\,ator, but you may have an advantage if you use a scientific \,or graphing calculator on some question\,s. Calculators permitted during testing are: ■ Graphing calculators ■ Scientific calculators ■ Four-function calculators (not recommended). See page 12 for the calculators that are permitted for use on the S\bT. Test-takers using a calculator with large characters (one inch high or more) or raised display that might be vis\,ible to other test- takers will be seated at the discretion of the test supervisor in a location where other test-takers cannot view the large or raised display. Unacceptable Calculators The following calculators are not permitted: ■ Portable/handheld computers, laptops, electronic writing pads, pocket organizers ■ Models that have a QWERTY (i.e., typewriter) keypad, either as part of hardware or software (e.g., TI-92 Plus, Voyage 200) ■ Models that have pen-input, stylus or touch-screen capability (e.g., Palm, PD\bs, Casio ClassPad) or have wireless or Bluetooth capability ■ Models that use pa\,per tape, “talk” or make unusual noises, o\,r require an electrical out\,let ■ Models that can access the Internet ■ Models that have cell phone capabilit\,y or have audio/video recording capability ■ Models that have a digital audio/video player or have a camera or scanning capab\,ility

12 Getting Ready for the SAT Four-fun ction calculators:  Although not recommended,  any four-function calculator is permitted.   Scientific calculators: An y scientific calculator \bprogra mmable or nonprogrammable) is permitted. Graphin\b calculators : The following \braphin\b calculators  are permitted. This list will be updated as needed .  s t n e m u r t s n I   s a x e T   rp a h S   d r a k c a P - t t e l w e H   o i s a C FX-6000 seri es  FX-6200 seri es  FX-6300 seri es  FX-6500 seri es  FX-7000 seri es  FX-7300 seri es  FX-7400 seri es  FX-7500 seri es  FX-7700 seri es  FX-7800 seri es  FX-8000 seri es  FX-8500 seri es  FX-8700 seri es  FX-8800 seri es    FX-9700 seri es  FX-9750 seri es  FX-9860 seri es  CF X-9800 series  CF X-9850 series  CF X-9950 series  CF X-9970 series  FX 1.0 series  Algebra FX 2.0 series    HP-9G   HP-28 series   HP-38G  HP-39 series   HP-40 series HP-48 series   HP-49 series   HP-50 series       Radio Shac k  EC-4033  EC-4034  EC-4037  EL-5200  EL-9200  series  EL-9300  series  EL-9600  series*  EL-9900  series            Othe r   Datexx DS-883  Micronta  Smart 2   TI-73  TI-80  TI-81  TI-82  TI-83/TI- 83 Plus  TI-83 Plus Silver   TI-84 Plus  TI-84 Plus Silver   TI-84 Plus C Silver  TI-85  TI-86  TI-89  TI-89 Titanium  TI-Nspire TI-Nspire CM-C TI-Nspire CX TI-Nspire CA S TI-Nspire CX CAS TI-Nspire CAS CX-C *The use of the sty\alus   is not permitted.\a The followin\b calcul\fiators are permitted\fi on the SA T: FX-CG-10 \bPRIZM) FX-CG-20

Getting Ready for the SAT 13 Mathematics Review Number and Operations (20–25%) ⦁ Arithmetic wor\b problems (inclu\bing percent, ratio an\b proportion) ⦁ Properties of integers (even, o\b\b, prime numbers, \bivisibility, etc.) ⦁ Rational numbers ⦁ Sets (union, intersection, elements) ⦁ Counting techniques ⦁ Sequences an\b series (inclu\bing exponential growth) ⦁ Elementary number theory \blgebra and Functions (35–40%) ⦁ Substitution an\b simplifying algebraic expressions ⦁ Properties of exponents ⦁ Algebraic wor\b problems ⦁ Solutions of linear equations an\b inequalities ⦁ Systems of equations an\b inequalities ⦁ Qua\bratic equations ⦁ Rational an\b ra\bical equations ⦁ Equations of lines ⦁ Absolute value ⦁ Direct an\b inverse variation ⦁ Concepts of algebraic functions ⦁ Newly \befine\b symbols base\b on commonly use\b operations Geometry and Measurement (25–30%) ⦁ Area an\b perimeter of a polygon ⦁ Area an\b circumference of a circle ⦁ Volume of a box, cube an\b cylin\ber ⦁ Pythagorean Theorem an\b special properties of isosceles, equilateral an\b right triangles ⦁ Properties of parallel an\b perpen\bicular lines ⦁ Coor\binate geometry ⦁ Geometric visualization ⦁ Slope ⦁ Similarity ⦁ Transformations Data \bnalysis, Statistics and Probability (10–15%) ⦁ Data interpretation (tables an\b graphs) ⦁ Descriptive statistics (mean, me\bian, mo\be an\b range) ⦁ Probability Number and Operations ⦁ Integers: . . . , −4, −3, −2, −1, 0, 1, 2, 3, 4, . . . (Note: zero is neither positive nor negative.) ⦁ Consecutive Integers: Integers that follow in sequence; for example, 22, 23, 24, 25. Consecutive integers can be more generally represente\b by n, n + 1, n + 2, n + 3, . . . ⦁ Odd Integers: . . . , −7, −5, −3, −1, 1, 3, 5, 7, . . . , 2 1 k + , . . . where k is an integer ⦁ Even Integers: . . . , −6, −4, −2, 0, 2, 4, 6, . . . , 2 k , . . . , where k is an integer (Note: zero is an even integer.) ⦁ Prime Numbers: 2, 3, 5, 7, 11, 13, 17, 19, . . . (Note: 1 is not a prime and 2 is the only even prime.) ⦁ Digits: 0, 1, 2, 3, 4, 5, 6, 7, 8, 9 (Note: the units digit and the ones digit refer to the same digit in a number. For example, in the number 125, the 5 is called the units digit or the ones digit.) Percent Percent means hun\bre\bths, or number out of 100. For example, 40 percent means 40 100 or 0.40 or 2 5. Prob\bem 1: If the sales tax on a $30.00 item is $1.80, what is the sales tax rate? So\bution: $ . $ . 1 80 100 30 00 = × n n= 6 6, % so is the sales tax rate. Percent Increase / Decrease Prob\bem 2: If the price of a computer was \becrease\b from $1,000 to $750, by what percent was the price \becrease\b? So\bution: The price \becrease is $250. The percent \becrease is the value of n in the equation 250 1 000 , = n 100 . The value of n is 25, so the price was \becrease\b by 25%. Note: n% increase means increase original = n 100 ; n% decrease means decrease ori ginal = n 100 .

14 Getting Ready for the SAT \bverage Speed Prob\bem: José travele\b for 2 hours at a rate of 70 kilome- ters per hour an\b for 5 hours at a rate of 60 kilometers per hour. What was his average spee\b for the 7-hour perio\b? So\bution: In this situation, the average spee\b was total distance total time The total \bistance was 2 hr 70 km hr      + 5 hr 60 km hr      = 440 km. The total time was 7 hours. Thus, the average spee\b was 440 7 km hr = 62 6 7 kilometers per hour. Note: In this example, the average spee\b over the 7-hour perio\b is not the average of the two given spee\bs, which woul\b be 65 kilometers per hour. Sequences Two common types of sequences that appear on the SAT are arithmetic an\b geometric sequences. An arithmetic sequence is a sequence in which successive terms \biffer by the same constant amount. For example: 3, 5, 7, 9, . . . is an arithmetic sequence. A geometric sequence is a sequence in which the ratio of successive terms is a constant. For example: 2, 4, 8, 16, . . . is a geometric sequence. A sequence may also be \befine\b using previously \befine\b terms. For example, the first term of a sequence is 2, an\b each successive term is 1 less than twice the prece\bing term. This sequence woul\b be 2, 3, 5, 9, 17, . . . On the SAT, explicit rules are given for each sequence. For example, in the sequence above, you woul\b not be expecte\b to know that the 6th term is 33 without being given the fact that each term is one less than twice the prece\bing term. For sequences on the SAT, the first term is never referre\b to as the “zeroth” term. Algebra and Functions Factoring You may nee\b to apply these types of factoring: x x x x2 2 2 + = + ( ) x x x2 1 11 − = + ( ) − ( ) x x x x x2 2 2 1 1 1 1 + + = + ( ) + ( ) = +( ) 2 5 3 21 3 2x x x x + − = − ( ) + ( ) Functions A function is a relation in which each element of the \bomain is paire\b with exactly one element of the range. On the SAT, unless otherwise specifie\b, the \bomain of any functio n f is assume\b to be the set of all real numbers fx( ) is a real number. For example, if fx x( ) = + 2 , the \bomain of f is all real numbers greater than or equal to −2. For this function, 14 is paire\b with 4, since f14 142 16 4( ) = + = = . Note: The symbol represents the positive, or principal, square root. For example, 16 4 = , not ±4. Exponents You shoul\b be familiar with the following rules for exponents on the SAT. For all values of a bx y , , , : x x xa b a b = + x xab a b( ) = x y x y a a a () = For all values of a bx y , , , : > >0 0 x xxa b a b= − x y x y a a a       = x x a a −= 1 Also, x x ab a b= . For example, x x 23 2 3= . Note: For any nonzero number x, it is true that x0 1 =. Variation Direct Variation: The variable y is \birectly proportional to the variable k such that y k x = . Inverse Variation: The variable k such that y k x x y k = = or . \bbsolute Value The absolute value of x is \befine\b as the \bistance from x to zero on the number line. The absolute value of x. For all real numbers x: x x x x x = ≥ − <  , , if if 0 0 For example: 2 2 2 0 2 2 2 2 0 0 0 = > − = − − = − =      , ( ) , si nc e sinc e < . . .

Getting Ready for the SAT 15 Geometry and Measurement Figures that accompany problems are inten\be\b to provi\be information useful in solving the problems. They are \brawn as accurately as possible EXCEPT when it is state\b in a particular problem that the figure is not \brawn to scale. In general, even when figures are not \brawn to scale, the rela - tive positions of points an\b angles may be assume\b to be in the or\ber shown. Also, line segments that exten\b through points an\b appear to lie on the same line may be assume\b to be on the same line. A point that appears to lie on a line or curve may be assume\b to lie on the line or curve. The text “Note: Figure not \brawn to scale” is inclu\be\b with the figure when \begree measures may not be accurately shown an\b specific lengths may not be \brawn proportion- ally. The following examples illustrate what information can an\b cannot be assume\b from figures. Example 1: Since AD an\b BE are line segments, angles DCE are vertical angles. Therefore, you can conclu\be that x y= . Even though the figure is \brawn to scale, you shoul\b NOT make any other assumptions without a\b\bitional informa- tion. For example, you shoul\b NOT assume that AC C D = or that the angle at vertex E is a right angle even though they might look that way in the figure. Example 2: Note: Figure not \brawn to scale. A question may refer to a triangle such as ABC above. Although the note in\bicates that the figure is not \brawn to scale, you may assume the following from the figure: ⦁ D is between A, D an\b C are points on a line. ⦁ The length of AD is less than the length of AC . ⦁ The measure of angle ABC. You may not assume the following from the figure: ⦁ The length of AD is less than the length of DC . ⦁ The measures of angles BAD an\b BD A are equal. ⦁ The measure of angle ABD is greater than the measure of angle DB C. ⦁ Angle  m k a° b° c° d° w° x° y° z° 1. If two parallel lines are cut by a thir\b line, the alternate interior angles are congruent. In the figure above, c= x and w =d . 2. If two parallel lines are cut by a thir\b line, the cor- respon\bing angles are congruent. In the figure, a = w, b= x, c = y, and d= z . 3. If two parallel lines are cut by a thir\b line, the sum of the measures of the interior angles on the same si\be of the transversal is 180°. In the figure, c w d x+ = + = 180180 and . \bngle Relationships 60° 50° x° y° z° 1. The sum of the measures of the interior angles of a triangle is 180°. In the figure above, x=70 because 60 50 180 + += x . 2. When two lines intersect, vertical angles are congruent. In the figure, y=50 . 3. A straight angle measures 180°. In the figure, z=130 because z+ =50 180 .

16 Getting Ready for the SAT 4. In any triangle, the longest si\be is opposite the largest angle, an\b the shortest si\be is opposite the smallest angle. In the figure below, a b c< < . 5. Two polygons are similar if an\b only if the lengths of their correspon\bing si\bes are in the same ratio an\b the measures of their correspon\bing angles are equal. If polygons AF an\b GL are correspon\bing si\bes, so that AF GL BC HI x = = = = 10 5 2 1 18 .Therefore, x HI= =9 . Note: AF means the line segment with en\bpoints A an\b AF . \brea and Perimeter Rectangles Area of a rectangle = lengt h w idth= × ×  w Perimeter of a rectangle Circles Area of a circle = r2 π (wher e r is the ra\bius) Circumference of a circle = 2πr=π d (wher e d is the \biameter) Triangles Area of a triangle = 1 2ba se altitud e ×( ) Perimeter of a triangle = the sum of the lengths of the three si\bes Triangle inequality: The sum of the lengths of any two si\bes of a triangle must be greater than the length of the thir\b si\be. 4. The sum of the measures of the interior angles of a polygon can be foun\b by \brawing all \biagonals of the polygon from one vertex an\b multiplying the number of triangles forme\b by 180°. Since this polygon is \bivi\be\b into 3 triangles, the sum of the mea - sures of its angles is 3  × × w h 180°, or 540°. Unless otherwise note\b in the SAT, the term “polygon” will be use\b to mean a convex polygon; that is, a polygon in which each interior angle has a measure of less than 180°. A polygon is “regular” if all its si\bes are congruent an\b all its angles are congruent. Side Relationships 1. Pythagorean Theorem: In any right triangle, a b c2 2 2+ = , wher e c is the length of the longest si\be an\b a an \b b are the lengths of the two shorter si\bes. To fin\b the value of x, use the Pythagorean Theorem. x x x x2 2 2 2 2 3 4 9 16 25 25 5= + = + = = = 2. In any equilateral triangle, all si\bes are congruent an\b all angles are congruent. Because the measure of the unmarke\b angle is 60°, the measures of all angles of the triangle are equal; therefore, the lengths of all si\bes of the triangle are equal: x y= = 10. 3. In an isosceles triangle, the angles opposite con- gruent si\bes are congruent. Also, the si\bes opposite congruent angles are congruent. In the figures below, a b x y= = and .

Getting Ready for the SAT 17 Volume Volume of a rectangular soli\b (or cube) = × × w h (  is the length, w is the wi\bth an\b h is the height) Volume of a right circular cylin\ber = r h2 π (r is the ra\bius of the base, an\b h is the height) Be familiar with the formulas that are pro\bided in the Reference Information included with the test directions. Refer to the test directions in the sample test in this publication. Coordinate Geometry 1. In questions that involve the x - an\b y -axes, x -values to the right of the y -axis are positive an\b x -values to the left of the y -axis are negative. Similarly, y -values above the x -axis are positive an\b y -values below the x -axis are negative. In an or\bere\b pair −2 an\b the y -coor\binate o f P is 3. Therefore, the coor\binates of poin t P are ( ) , −2 3 . Similarly, you can conclu\be that the line shown in the figure passes through the point with coor\binates ( ) , − −2 1 an\b the point =change in-coor dina tes chan ge i n - coor din y x a at es Sl ope ofPQ = = 4 2 2 Sl ope of= − − − − = − 1 2 2 2 3 4 ( ) A line that slopes upwar\b as you go from left to right has a positive slope. A line that slopes \bown- war\b as you go from left to right has a negative slope. A horizontal line has a slope of zero. The slope of a vertical line is un\befine\b. Parallel lines have the same slope. The pro\buct of the slopes of two perpen\bicular lines is −1 , provi\be\b the slope of each of the lines is \befine\b. For example, any line perpen\bicular to line  above has a slope of 4 3. The equation of a line can be expresse\b as ymx b= + , where m is the slope an\b b is the  is −3 4, the equation of line  can be expresse\b as y x b = − + 3 4 . Since the point x y= − = 2 1 an d must satisfy the equation. Hence, 1 3 2 1 2 = + = − b b , s o an\b the equa tion of line  is y x = − − 3 4 1 2. 3. A qua\bratic function can be expresse\b as ya x h k = − ( ) +2 where the vertex of the parabola is at the point ( , )h k an\b a 0≠ . If a> 0 , the parabola opens upwar\b; an\b if a

18 Getting Ready for the SAT \bata Analysis, Statistics and Probability Measures of Center An average is a statistic that is use\b to summarize \bata. The most common type of average is the arithmetic mean. The average (arithmetic mean) of a list of n numbers is equal to the sum of the numbers \bivi\be\b by n . For example, the mean of 2, 3, 5, 7 an\b 13 is equal to 2 3 5 713 5 6 + + + + =. When the average of a list of n numbers is given, the sum of the numbers can be foun\b. For example, if the average of six numbers is 12, the sum of these six numbers is The median of a list of numbers is the number in the mi\b- \ble when the numbers are or\bere\b from greatest to least or from least to greatest. For example, the me\bian of 3, 8, 2, 6 an\b 9 is 6 because when the numbers are or\bere\b, 2, 3, 6, 8, 9, the number in the mi\b\ble is 6. When there is an even number of values, the me\bian is the same as the mean of the two mi\b\ble numbers. For example, the me\bian of 6, 8, 9, 13, 14 an\b 16 is the mean of 9 an\b 13, which is 11. The mode of a list of numbers is the number that occurs most often in the list. For example, 7 is the mo\be of 2, 7, 5, 8, 7 an\b 12. The list 2, 4, 2, 8, 2, 4, 7, 4, 9 an\b 11 has two mo\bes, 2 an\b 4. Note: On the SAT, the use of the wor\b average refers to the arithmetic mean an\b is in\bicate\b by “average (arithmetic mean).” An exception is when a question involves average spee\b (see page 14). Questions involving me\bian an\b mo\be will have those terms state\b as part of the question’s text. Range The range of a list of numbers is the value obtaine\b by subtracting the smallest number in the list from the greatest number in the list. For example, in the list 8, 15, 11, -4, 0, 6, -7 an\b 12, the range is equal to the \bifference between 15 an\b -7, which is 22. Probability Probability refers to the chance that a specific outcome can occur. When outcomes are equally likely, probability can be foun\b by using the following \befinition: number of ways tha\lt a specif ic outcome can occ\lur total number of pos\lsible outcomes For example, if a jar contains 13 re\b marbles an\b 7 green marbles, the probability that a marble selecte\b from the jar at ran\bom will be green is Note: The phrase at random in the prece\bing example means that each in\bivi\bual marble in the jar is equally likely to be selecte\b. It \boes not mean the two colors are equally likely to be selecte\b. If a particular outcome can never occur, its probability is 0. If an outcome is certain to occur, its probability is 1. In general, i f p is the probability that a specific outcome will occur, values o f p fall in the range 0 1≤ ≤ p . Probability may be expresse\b as either a \becimal, a fraction or a ratio.

Getting Ready for the SAT 19 to use again. Most problems can be solved in a variety of ways, so don’t be concerned if your method is different from the one given. Note that the directions indicate that you are to select the \best of the choices given. Multiple-Choice Questions The questions that \,follow will give you an idea of the t\,ype of mathematical thinkin\,g required to solve problems on the S\bT. First, try to answer each question yourself, and then read the solutions that follow. These solutions may give you new insights into solving the problems or point out\, techniques you’ll be able \birections For this section, sol\be each problem and decide which is the best of the choices gi\ben. Fill in the corresponding circle on the answer sheet. You may use any a\bailable space for scratch work. Notes 1. The use of a ca\mlculator is permitt\med.  2. All num\bers used a\mre real num\bers. 3. Figures that ac\mcompany pro\blems in t\mhis test are intende\md to provide informa\mtion useful in solv\ming the pro\blems.      They are drawn \mas accurately as p\mossi\ble EXCEPT when \mit is stated in a s\mpecific pro\blem tha\mt the figure is not\m      drawn to scale.\m All figures lie in\m a plane unless oth\merwise indicated. 4. Unless otherwise\m specified, the doma\min of any function    f  is assumed to \be the\m set of all real nu\mm\bers  x for which      f(x) is a real num\ber. h r r h b   A =     r 2 C = 2  r  A =     bh V =  wh V =   r 2h  The number of degr\lees of arc in a ci\lrcle is 360. The sum of \bhe meas\lures in degrees of\l \bhe angles of a \bri\langle is 180. b ac c2 = a 2 + b 2  Special Righ\b Tria\lngles x   3 2x x 60° 30° s s45° 45° A =   w w w h Reference  Informa\bion 2 s 1 2 Sample Questions Below are seven examples of standard multiple-choice questions. Following each question,\, you will find one or\, two solutions. 1. \b special lottery is to be held to select the student who will live in the only deluxe room in a dormitory. There are 100 seniors, 150 juniors, and 200 sophomor\,es who applied. Ea\,ch senior’s name is placed in the lottery 3 times; each j\,unior’s name, 2 times; and each s\,ophomore’s name, 1 time. If a \,student’s name is chosen at r\,andom from the names in the\, lottery, what is the probability that a senior’s name will be chos\,en? (\b)  1 8 (B)  2 9 (C)  2 7 (D)  3 8 (E)  1 2 Correct answer: (D) / Difficulty level: Medium To determine the probability that a senior’s name will be chosen, you must determine the total number of senio\,rs’ names that are in the lottery and divide this\, number by the total number of names in t\,he lottery. Since each senior’s name is placed in the lottery 3 times, there are 3 100 300 × = seniors’ names. Likewise, there are 2 150 300 × = juniors’ names and 1 200 200 × = sophomores’ names in the lo\,ttery. The probability that a senior’s name will be chos\,en is 300 300 300 200 300 800 3 8 + + = = .

20 Getting Ready for the SAT NOONTIME TEMPER\bTURES IN HILO, H\bW\bII Mon. Tue.Wed. Thu. Fri.Sat. Sun. 66 787569 787770 2. The table above shows the temperatures at noon, in degr\,ees Fahrenheit, in a city in Hawaii over a one-week period. If m represents the median o\,f these temperatures, f represents the temperature that occurred most often, and a represents the average (arithmetic mean) \,of these seven temperatures, which of the following is the correct order of m, f, and a ? (\b) a m f < < . 3. The projected sales volume of a video game cartridge is given by the function s p p a( ) =+ 3000 2 , where s is the number of cartridges sold, in thousan\,ds; p is the price per cartridge, in dollars; and a is a constant. If according to the projections, 100,000 cartridges are sold at $10 per \,cartridge, how many cartridges will be sold at\, $20 per cartridge? (\b) 20,000 (B) 50,000 (C) 60,000 (D) 150,000 (E) 200,000 Correct answer: (C) / Difficulty level: Medium For 100,000 cartridges sold at $10 per\, cartridge, s = 100 (since s is the number of ca\,rtridges sold, in thousands ) and p = 10. Substituting into the equation yie\,lds 100 3000 2 10 =( ) + a . Solving this equa\,tion for a yields s pp( ) =+ 3000 2 1 0 . To determine how many cartridges will be sold at $20 per\, cartridge, you need to evaluate s 20 3000 2 20 1060 ( ) = ( ) += . Since s is given in thousands, there will be 60,000 cartridges sold at $20 per\, cartridge. x (1, 2) y O 4. In the xy-coordinate plane above, line  contains the points\, (0, 0) and (1, 2). If line m (not shown) contains the point \,(0, 0) and is perpendicula\,r to , what is an equati\,on of m ? (\b) y x = − 1 2 (B) y x = − + 1 2 1 (C) y x = − (D) y x = − +2 (E) y x = − 2 Correct Answer: (A) / Difficulty level: Medium Using the coordinates of the two points given on line , the slope of  is 2 0 1 0 2 − − = . Line m, which is perpendi\,cular to , will have a slope of −1 2, since slopes of perpend\,icular lines are negative reciprocals of each other. \bn equation of m can be written as y x b = − + 1 2 . Since line m also contains point (0, 0), it follows that b = 0. Therefore, an equation of line m is y x = − 1 2 .

Getting Ready for the SAT 21 Note:  Figure not \.drawn to scale. a b c 5. If two sides of the tria\,ngle above have lengths 5 and 6, the perimeter of the triangle \,could be which of the\, following? I. 15 II. 20 III. 22 (\b) I only (B) I and II only (C) I and III only \, (D) II and III only (E) I, II, and III Correct answer: (B) / Difficulty level: Hard In questions of thi\,s type, statements I, II, and II\,I should each be considered independently of the others. In this questio\,n, you must determine which of thos\,e statements could be true. ■ Statement I states that 15 could be the perimet\,er of the triangle. This is true. If the\, perimeter of the triangle \,is 15, and two sides have lengths 5 and 6, then t\,he third side of the triangle would have length 15 − (6 + 5), or\, 4. \b triangle can \, have side lengths of 4, 5, and 6. \,So the perimeter of the triangle could be 15. ■ Similarly, statement II is true. I\,f 20 is the perimeter of the triangle, then the \,third side of the trian\,gle would have length 20 – (6 + 5), or 9. \b triangle can ha\,ve side lengths of 5, 6, and 9. So the perimeter of the triangle \,could be 20. ■ Finally, consider whether th\,e triangle could have a perimeter of 22. In this case, th\,e length of the third side would be 22 – (6 + 5) = 11. \,The triangle inequa\,lity states that the sum of the lengths of any two sides of a triang\,le must be greater than the length of the third side. Since the sum of 5 and 6\, is not greater than 11, it follows that 5, 6, and 11\, cannot be the lengths of the sides of \,a triangle, and so \,the given triangle cannot have a perimeter of 22. Therefore, the correct answer to the question is \,I and II only, which is choice (B). 6. If x x x x m > = 1 3 and , what is the value of m ? (\b) −7 2 (B) −5 2 (D) −3 2 Correct answer: (C) / Difficulty level: Medium Since x can be written as x 1 2 and 13x can be written as x−3, the left side of the equat\,ion is x x x x x 1 2 1 2 5 2 5 2 3 3 ⋅ = = −−             − − . Since = =x m Since x x x x x 1 2 1 2 5 2 5 2 3 3 ⋅ = = −−             − − . Since = =x m, the value of m is − 52. 7. If k is divisible by 2, 3, and 15, whic\,h of the following is also divisible by these numbers? (\b) k + 5 (B) k+15 (C) k + 20 (D) k+ 30 (E) k+ 45 Correct answer: (D) / Difficulty level: Medium Since k is divisible by 2, 3, and 15, k must be a multiple\, of 30, as 30 is the least common multiple of 2, \,3, and 15. Some mult\,iples of 30 are 0, 30, 60, 90, and 120. ■ If you add two multiples of 30, the sum will also be a multiple of 30. For example, 60 and 90 are multiples of 30, and their sum, 150, is also a multiple of 30. ■ If you add a multiple o\,f 30 to a number that is \,not a multiple of 30, the sum will not be a multiple of 30\,. For example, 60 is a multiple of 30 \,and 45 is not. Their sum, 105, is no\,t a multiple of 30. ■ The question asks which answer choice is divisible by 2, 3, and 15 — that is, w\,hich answer choice is a multiple of 3\,0. \bll the answer choices are in the form of “k plus a number.” Only choice (D), k+ 30 , is the sum of k and a multiple of 3\,0. The sum of k and 30 is also a \,multiple of 30, so the correct answer is choice (D).

22 Getting Ready for the SAT Student-Produced Response Questions Questions of this type have no answer choices provided. Instead, you must solve the problem and fill in your answer on a special grid. Ten questions on the test will be of this type. It is very important for you to understand the directions for entering answers on the grid\b You will lose valuable testing time if you have to stop to figure out how to grid the answers when you take the test. Take the time now to understand how to enter your answers in this special format. \b primary advantage of this format is that it al\,lows you to enter the form of the answer that you obtain, whether \,a whole number, decimal or fraction. For example, if you obtain 2/5, you can grid 2/5. If you obtain .4, you can grid .4. Generally, you should grid the form of the answer that you obtain naturally in solving the\, problem. The grid will only \,hold numbers that range from 0 to 9999. Decimals and fractions can also be\, gridded. ■ Do your best to be certain of your answer before you grid it\b If you erase your answer, do so completely. Incomplete erasures may be picked up by the scoring machines as intended answers. ■ Check your work if your answer does not fit on \Wthe grid\b If you obtain a negative value, a value greater than 9999 or an\, irrational number, you have made an error. ■ Make an educated guess if you don’t know the answer\b On student-produced response (grid-in) \,questions, you don’t lose points for wrong answers. ■ Always enter your answer by filling in the ci\Wrcles on the grid\b Only answers entered on the grid are scored. Your handwritten answer at the top of the grid isn’\,t scored. However, writing your answer at the top of the grid may help you avoid gridding errors. Approaches to Student-Produced Response Questions ■ Decide in which column you want to begin gridding y\Wour answers before the test starts\b You can start gridding your answers in any column (space permitting), but it helps to decide on a method \,ahead of time. ■ If the answer is zero, grid it in column 2, 3 or 4\b Note that there is no circle to grid for zero in column 1. ■ Do not use leading\W zeros\b For example, if your answer is .125, you must grid .125 or convert it to the fraction 1/8. ■ A fraction does not ha\Wve to be reduced unless it will not fit the grid\b For example, 15/25 will not fit. The reduced form of 3/5 can be entered. Note that 6/10 and 9/15 would also be considered correct and do not have to be reduced. The decimal form, .6, can also be grid\,ded. Each of the remain\ling questions requ\lires you to solv e the problem and en\lter your ans\ber by \lmarking the circles\l in the special grid\l, as sho \bn in the examples belo \b. You may use an y available space for sc\lratch \bork. Decimal Answers:  If you obtain a d\lecimal ans\ber  \bith more digits th\lan the grid can acc\lommodate,  it may be either ro\lunded or truncated, \lbut it must  fill the entire gri\ld. For example, if \lyou obtain  an ans\ber such as 0.\l6666..., you should\l record  your result as .666\l or .667. A less accurate va\.lue  such as .66 or .67 wi\.ll be score\b as inc\.orrect. Acceptable \bays to g\lrid     are:    2 3 Note:  You may start you\lr ans\bers in any column, spac\le permitting. Columns not needed s\lhould be left blank. Mark no more than \lone circle in any c\lolumn. Because the ans\ber \lsheet \bill be machin\le- scored, you will receive c\.re\bit only if the c\.ircles  are fille\b in corre\.ctly. Although not requir\led, it is suggested\l that you \brite your ans\ber i\ln the boxes at the \ltop of the columns to help you\l fill in the circles\l accurately.  Some problems may h\lave more than one c\lorrect ans\ber. In such cas\les, grid only one a\lns\ber. No question has a \lnegative ans\ber. Mixe\b numbers such as 3    must \lbe gridded as 3.5 or 7  2. (If            \l      is gridded, it\l \bill be interpreted as     \l    , not 3   .) 1 2 1 2 31 2 • • • • • • 1 2 3 4 5 6 0 1 2 3 4 5 60 1 2 3 4 5 60 1 2 3 4 5 6 1 2 3 4 5 6¥ 0 1 2 3 4 5 0 1 2 3 4 50 1 2 3 4 5 1 2 3 4 5 6¥ 0 1 2 3 4 5 0 1 2 3 4 50 1 2 3 4 5 6 Answer:   2.5 Fraction line Decimal point Write answer in bo\bes. Grid in result. Answer:   201 Either  position  is correct. 7 12 1 2 3 4 5 6 7 8 9 0 1 2 3 4 5 6 7 8 90 1 2 3 4 5 6 7 8 90 1 2 3 4 5 6 7 8 9 1 2 3 4 5 6 7 8 90 1 2 3 4 5 6 7 8 90 1 2 3 4 5 6 7 8 90 1 2 3 4 5 6 7 8 9 1 2 3 4 0 1 2 3 40 1 2 3 4 0 1 2 3 4 0 1 2 3 40 1 2 3 0 1 2 3 1 2 3 Answer: • Below are the actual directions that you will find on the\, test — read them carefully.

Getting Ready for the SAT 23 9. For all positive integers a and b, let a ♦ b be defined by a ♦ b = + − a a b 1 1. What is the value of 4 ♦ 2? 1 2 3 4 5 6 7 8 9 0 1 2 3 4 5 6 7 8 90 1 2 3 4 5 6 7 8 90 1 2 3 4 5 6 7 8 9 7 1 3 / 1 2 3 4 5 6 7 8 90 1 2 3 4 5 6 7 8 90 1 2 3 4 5 6 7 8 90 1 2 3 4 5 6 7 8 9 6 5 6 1 2 3 4 5 6 7 8 90 1 2 3 4 5 6 7 8 90 1 2 3 4 5 6 7 8 90 1 2 3 4 5 6 7 8 9 7 5 6 The words “let a ♦ b be defined by ” tell you that the symbol ♦ is not supposed to represent a common mathematical op\,eration, but one that is mad\,e up for this question. \,To evaluate 4 ♦ 2, you substitute 4 for a and 2 for b in the expression a ab+ − 1 1 . This gives 4 1 4 1 2+ − , which equals 17 3. The answer may be entered in the grid as \,17/3 or as 5.66 or 5.67. Difficulty level: Medium 10. Of the 6 courses offered by the music department at her college, Kay must choose exactly 2 of them. How many different combinations of 2 courses are possible for Kay if there are no restrictions on whic\,h 2 courses she can choose? 1 2 3 4 5 6 7 8 0 1 2 3 4 5 6 7 80 1 2 3 4 5 6 7 80 1 2 3 4 5 6 7 8 9 9 9 9 5 1 There are 6 courses offered; let us refer to them as 1, 2, 3, 4\,, 5 and 6. One way to find the number of \,combinations is to list all possible pairings. They are 1-2, 1-3, 1-4, 1-5, 1-\,6, 2-3, 2-4, 2-5, 2-6, 3-4, 3-5, 3-6, 4\,-5, 4-6 and 5-6. There are 15 combinations. Note that 1-2 and 2-1 represent the same combination, so only\, one is in the list\,. Sample Questions Below are five examples of student-produced response questions. Following each question,\, you will find a solu\,tion and several ways to enter the correct answer. 4 7 5 3 8 1 x x − = − = 8. What value of x satisfies both of th\,e equations above? 1 2 3 4 5 6 7 8 9 0 1 2 3 4 5 6 7 8 90 1 2 3 4 5 6 7 8 90 1 2 3 4 5 6 7 8 9 1 2 / 1 2 3 4 5 6 7 8 90 1 2 3 4 5 6 7 8 90 1 2 3 4 5 6 7 8 90 1 2 3 4 5 6 7 8 9 5 Since 4 7 5x− = , the value of 4 7 5 5 x − − iseither or . 4 7 5 4 2 12 x x x − = − = = The two values of x that satisfy the fi\,rst equation are 3 and 1 2. x x 3 81,                \        3 8 1− =−− 1. Since the value of is either  or 3 81 8 2 1 4 − = = = x x x or 3 8 1 8 4 12 − = − = = x x x The two values of x that satisfy the s\,econd equation are 1 4 and 1 2. You are asked to find the value of x that satisfies both equations. That value is 1 2. The answer can be entered in the grid as \,1/2 or .5. Difficulty level: Hard : :

24 Getting Ready for the SAT You could also notice that there are 5 pairings that \,start with course 1 and 4 additio\,nal pairings that \,start with course 2, and so forth. The total number of combinations is 5 4 3 21 15+ + + += . You could also solve the problem by noting that the \,total number of permutations (tha\,t is, the number of \,different ways 2 of 6 courses could be selected) is 6 for the first course selected times 5 for the second course selected, or 6 5 30 × = . To find the number of combinations, you must divide the \,number of permutations by the number of arrangements. For each pair of courses A-B selected, the arrangement B-A is also possible. Therefore, there are 2 arrangements. So the numb\,er of combinations is 30 2 1 5 ÷ =. Difficulty level: Medium 11. Let the function what is one possible value ofbe defined by f fx x x ( ) = − 2 7 + + + ( ) = 10 1 0 . , If f t t ? 1 2 3 4 5 6 7 8 90 1 2 3 4 5 6 7 8 90 1 2 3 4 5 6 7 8 90 1 2 3 4 5 6 7 8 9 1 1 2 3 4 5 6 7 8 90 1 2 3 4 5 6 7 8 90 1 2 3 4 5 6 7 8 90 1 2 3 4 5 6 7 8 9 4 Since f t t t t +( ) = + +( ) − +( ) + 1 2 1 77 10 2 , or ft+( ) = 1 0 , it follows that k t= + 1. Then k t= + 1and k k= = 5 2or . Since Sinc e or t k t t = − = = 1 4 1 , . it follows that Si nc e o r t k t t = − = = 1 4 1 ,. This question asks for one possible value of t. Either 1 or 4 satisfies the quest\,ion being asked. Choose only one correct answer (not both) to enter in the grid. When there is a range of possible correct answers, your gridded response must lie within th\,e range. For example, consider a problem for which all number\,s between 4 and 5, exclusive, are correct answers. For this problem, although 4.0002 is within the range 4 5<

Getting Ready for the SAT 25 Note: Calculators may not be on your desk or used o\Wn the writing secti\Won of the SAT\b The Writing Section The writing section \,includes both multi\,ple-choice questions and a \, direct writing measure in the form of an essay. The multiple-choice sections include:\, ■ Improving sentences (25 questions) ■ Identifying sentence errors (18 questions) ■ Improving paragraphs (6 questions) The multiple-choice sections measure your ability to ■ communicate ideas clearly and\, effectively; ■ improve a piece of writing through revision and editing;\, ■ recognize and identify sent\,ence-level errors; ■ understand grammatical elements a\,nd structures and how they relate to one another in a\, sentence; ■ recognize correctly formed grammatical structures; ■ clearly express ideas through sentence combining and use of \, transitional words and phrases; and ■ improve coherence of ideas within a\,nd among paragraphs. Characteristics of Effective Writing Multiple-choice writing questions focus on common problems associated with four characteristics of effecti\be writing. Illustrations of problems are gi\ben below. Multiple-choice writing questions also require recognition of correct sentences and effecti\be writing strategies. Writing problem Sentence illustrating the problem Should be\b\b\b 1. Being consistent Sequence of tenses After he broke his arm, he is home for two weeks. After he broke his arm, he was home for two weeks. Shift of pronoun If you are tense, one should try to relax. If you are tense, you should try to relax. Parallelism She skis, plays tennis, and flying hang gliders. She skis, plays tennis, and flies hang gliders. Noun agreement Carmen and Sarah are both a pilot. Carmen and Sarah are both pilots. Pronoun reference Se\beral people wanted the job, so he or she filled out the required applications. Se\beral people wanted the job, so they filled out the required applications. Subject–\berb agreement There is eight people on the shore. There are eight people on the shore. 2. Expressing ideas logically Coordination and subordination Tawanda has a rash, and she is probably allergic to something. Tawanda has a rash; she is probably allergic to something. Logical comparison Nathan grew more \begetables than his neighbor’s garden. Nathan grew more \begetables than his neighbor grew. Modification and word order Barking loudly, the tree had the dog’s leash wrapped around it. Barking loudly, the dog wrapped its leash around the tree. 3. Being clear and precise Ambiguous and \bague pronouns In the newspaper they say that few people \boted. The newspaper reported that few people \boted. Diction He circum\bented the globe on his trip. He circumna\bigated the globe on his trip. Wordiness There are many problems in the contemporary world in which we li\be. There are many problems in the contemporary world. Improper modification If your car is parked here while not eating in the restaurant, it will be towed away. If you park here and do not eat in the restaurant, your car will be towed away. 4. Following conventions Pronoun case He sat between you and I at the stadium. He sat between you and me at the stadium. Idiom Natalie had a different opinion for her. Natalie had a different opinion of her. Comparison of modifiers Of the sixteen executi\bes, Naomi makes more money. Of the sixteen executi\bes, Naomi makes the most money. Sentence fragment Fred ha\bing to go home early. Fred has to go home early. Comma splice Mary took time out of her busy schedule to \bisit her aunt, John decided to continue working through the summer. Mary took time out of her busy schedule to \bisit her aunt, but John decided to continue working through the summer. 5. Recognizing effective writing Some sentences require students to recognize that there is no error.

26 Getting Ready for the SAT Improving Sentences This question type measures your ability to ■ Recognize and correct faults in grammar and sentence structure. ■ Recognize effective sentences that follow the conventions of standard written English. \birections The following sentence tests correctness and effecti\beness of expression. Part of the sentence or the entire sentence is underlined; beneath the sentence are fi\be ways of phrasing the underlined material. Choice A repeats the original phrasing; the other four choices are different. If you think the original phrasing produces a better sentence than any of the alternati\bes, select choice A; if not, select one of the other choices. In making your selection, follow the requirements of standard written English; that is, pay attention to grammar, choice of words, sentence construction, and punctuation. Your selection should result in the most effecti\be sentence — clear and precise, without awkwardness or ambiguity. EXAMPLE: Laura Ingalls Wilder published her first book and she was sixty-fi\be years old then . (A) and she was sixty-fi\be years old then (B) when she was sixty-fi\be (C) at age sixty-fi\be years old (D) upon the reaching of sixty-fi\be years (E) at the time when she was sixty-fi\be a , c d e Answering Improving Sentences Questions Read the entire sentence carefully but quickly a\,nd ask yourself whether the underl\,ined portion is correct or whether it n\,eeds to be revised. Remember that choice (\b) is the same as th\,e underlined portion. Even if you think that the \,sentence does not require correction and that choice (\b) is the correct answer, it is a good idea to read each choice quickly to make sure. When reading choices (\b) through (E), replace the underlined part of the sentence with each answer choice to determine which revision results in a sentence that is clear and\, precise and meets the requirements of standard written English. Look for common problem areas in sentences. These include subject–verb agreement, parallelism, placement of modifiers, and the use of relative clauses. Remember that the ri\,ght answer will be the one\, correct version among the five choices. Keep in mind that wh\,ile the answer choices change, the rest of the sentence stays the same. In the example, connecting the two ideas (“Laura Ingalls Wilder published her first book”) and (“sh\,e was sixty-five years old then”) with the word “and” indicates that the two ideas are independent and eq\,ually important. The word “and” should be replaced to establish the relationship between the two ideas. ■ The word “and” indicates that the two ideas it connects are independent and eq\,ually important. No. ■ Replacing the word “and” with “when” clearly expresses the information that the sentence is intended to convey by relating Laura Ingalls Wilder’s age to her achievement. Yes, but continue to look at the other revisions. ■ Using the word “at” results in a phrase that is not id\,iomatic. No. ■ The phrase “upon the reaching of” also results in a phrase that is not idiomatic. N\,o. ■ The phrase “at the time when sh\,e was sixty-five” is awkward and wordy. No. Correct answer: (B) / Difficulty level: Easy Sample Question 1. Looking up from the base of the m\,ountain, the trail seemed more treacherous than it really was. (\b) Looking up (B) While looking up\, (C) By looking up (D) Viewing (E) Viewed When a modifying phr\,ase begins a sentence, it must logically\, modify the sentence’s subject; otherwis\,e, it is a dangling modifier. In this example, every option except (E) is a dangl\,ing modifier. ■ In (\b), the phrase “Looking up from the base of the m\,ountain” does not logically \,modify the subject “\,the trail.” \b person might stand at the \,base of a mountain a\,nd look up at a tr\,ail, but it is illogical\, to suggest that a trail looks up from the base of a mountain. ■ (B), (C) , and (D) are simply variations of the er\,ror found in (\b). Each results in a sentence that illogically \,suggests that a trail was looking up from the base of a mou\,ntain. ■ (E) is correct. \blthough a trail cannot itself lo\,ok up from the base of a mountain, \,a trail can be viewed by someone looking up from the base of a mou\,ntain, so the phrase “Viewed from the base of the moun\,tain” logically mod\,ifies the subject “the trail.” Correct answer: (E) / Difficulty level: Hard

Getting Ready for the SAT 27 ■ The phrase “The other” correctly modifies the w\,ord “delegates.” ■ The pronoun “him” is in t\,he wrong case. (One would not say “him immediately accepted.”) “Him” is an error, but go on to check the other choi\,ces, especially if you are not sure. ■ The word “immediately” correctly modifies the verb “accepted.” ■ The phrase “drafted by” correctly expresses the action of th\,e “neutral states.” Correct answer: (B) / Difficulty level: Easy Sample Question 2. \bfter hours of futile debate, the committee has \b decided to postpone further discussion B of the resolution until their next meeting. C D No error E ■ The error in this sentence occurs at (D). \b pronoun must agree in number (singular or plural) with the noun to which it refers. Collective nouns such as “committee“ can be treated as plural when members of the group act as individuals, or as singular when the group functions as a unit. In this sentence, the committee acts as a singular unit, as shown by the use of the singular verb “has,” which is not underlined and so cannot be changed. Therefore, the plural pronoun “their“ at (D) is used incorrectly. ■ The other options contain no errors. In (\b), the preposition “\bfter” appropriately introduces a phrase that indicates when the committee made its decisio\,n. In (B), “to postpone” is the verb form needed to complete the description o\,f the committee’s decision. In (C), the prepositional phrase “of the resolution” appropriately specifies the su\,bject of the postponed discussion. ■ The sentence may be corrected as follows: \bfter hours of futile debate, the committee has decided to postpone further discussion of the resolution until its next meeting. Correct answer: (D) / Difficulty level: Hard Improving Paragraphs This type of question meas\,ures your ability to ■ Edit and revise sentences in the context of a paragraph or entire essay. ■ Organize and develop paragraphs in a coherent and logical manner. ■ \bpply the conventions of standard written English. Identif\bing Sentence Errors This question type measures your ability to ■ Recognize faults in grammar and usage. ■ Recognize effective sentences that follow the conventions of standard written English. \birections The following sentence tests your ability to recognize grammar and usage errors. The sentence contains either a single error or no error at all. No sentence contains more than one error. The error, if there is one, is underlined and lettered. If the sentence contains an error, select the one underlined part that must be changed to make the sentence correct. If the sentence is correct, select choice E. In choosing answers, follow the requirements of standard written English. EXAMPLE: The other delegates and him immediately A B C accepted the resolution drafted by the D neutral states. No error E a , c d e Answering Identif\bing S\hentence Errors Questions Read the entire sentence carefully but quickly, paying attention to the underlined ch\,oices (\b) through (D). \bsk yourself whether any of the underlined\, words or phrases in the sentence contains a grammar or usage error. Keep in mind that so\,me sentences do not contain an error. Select the underlin\,ed word or phrase that must be ch\,anged to make the sentence correct. Mark (E) No error if you believe that the sentence is correct as written. Develop the habit of looking for the most common mistakes that people make in grammar: subject–verb agreement, pronoun reference and agreement, and adjective/adverb confusion. In the example above, “The other delegates and him” are the people who “immediately accepted the resolution,” and the phrase “drafted by the neutral states” describes “the resolution.” Check each underlined word or phrase for correctness.

28 Getting Ready for the SAT \birections The following passage is an early draft of an essay. Some parts of the passage need to be rewritten. Read the passage and select the best answers for the questions that follow. Some questions are about particular sentences or parts of sentences and ask you to impro\be sentence structure or word choice. Other questions ask you to consider organization and de\belopment. In choosing answers, follow the requirements of standard written English. Answering Improving Paragraphs Questions To answer the improving paragraphs questions tha\,t accompany the draft essay, you will need to note what sentences need to be corrected and to know how each sentence relates to the other sentences and to the essay as a whole. Read the entire essay quickly to determine its overall meaning. The essay is intended as a draft, so you will notice errors. In answering each question\,, make sure that your answer about a particular sentence or group of sentences makes sense in the context of the passage as a whole. Choose the best ans\,wer from among the choices given, even if you can imagine anot\,her correct response. Sample Questions Questions 3 and 4 are based on the following passage: (1) Many times art history courses focus on the great “masters,” ignoring those w\,omen who should have achieved fame. (2) Often women artists like Mary Cassatt have worked in the shadows of their male contemporaries. (3) They have rarely received much attention during thei\,r lifetimes. (4) My art teacher has tried to make up for it by teaching us about women artists and their work. (5) Recently she came to class very excited; she had just r\,ead about a little\,- known artist named \bnnie Jo\,hnson, a high schoo\,l teacher who had lived all of her life in New Haven, Connecticut. (6) Johnson never sold a painting\,, and her obituary \,in 1937 did not even mention her many paintings. (7) Thanks to Bruce Blanchard, a Connecticut business- man who bought some of her watercolors at an estate sale. (8) Johnson is finally starting to get the attention that she deserved more than one hundred years ago. (9) Blanchard now owns a private collection of hundreds of Johnson’s works — watercolors, charcoal sketches, and pen-and-ink drawings. (10) There are portraits and there are landscapes. (11) The thing that makes her work stand out are the portraits. (12) My teacher described th\,em as “unsentimenta\,l.” (13) They do not idealize characters. (14) Characters are presented almost photographically. (15) Many of the people in the pictures had an isolated, haunted look. (16) My teacher said that is\,olation symbolizes Johnson’s life as an artist. 3. In context, which is the be\,st revision to the underlined p\,ortion of sentence 3 (reproduced below)? They have rarely received much attention during their l\gifetimes. (\b) In fact, they had (B) Too bad these artists have (C) \bs a result, these women have (D) In spite of this, women artists (E) Often it is the case t\,hat the former have \blthough sentence 3 is not grammatically incorrect, its relationship to the preceding sentence needs to be made clearer. \b transitional phrase should be adde\,d to emphasize the cause- and-effect relationship between the stated facts — women artists received little attention as a consequence of having worked in the shadows of their male contemporaries — and the amb\,iguous pronoun “They” should be replaced with a word or phrase that clearly refers to the “women artists” and not the\, “male contemporaries” mentioned in\, sentence 2. ■ (\b), (B), and (D) are unsatisfactory because in each\, case the transitional phrase (“In fact,” “Too bad,” or “In spite of this”) fails to indicate the cause-and-effect relationship. Moreover, both (\b) and (B) leave the ambiguity of the pronoun unresolved. ■ (E) is unsatisfactory not only becaus\,e it fails to signal the cause-and-effect relationship, but also because \,it is wordy and illogically combines the adverbs “Often” and “rarely.” ■ (C) is correct. The transitional phrase “\bs a result” clearly indicates a cause-and-effect relationship, and “these women” properly resolves the ambiguity of the pronoun “They.” Correct answer: (C) / Difficulty level: Hard 4. In context, which of the following is the best version of sentence 10 (reproduced below)? There are portraits and there are landscapes. (\b) (\bs it is now) (B) You can see both por\,traits and landscapes\,. (C) Therefore, both portraits and landscapes\, are among her works. (D) Johnson painted both portraits and landscapes. (E) \bmong them Johnson has portraits and landscapes. In addition to being vague, sentence 10 contains no noun to which the pronoun “her” in sen\,tence 11 may refer. It should be revised so that Johnson is\, clearly identified \,as the painter of the portraits and landscapes.

Getting Ready for the SAT 29 ■ (\b), (B), and (C) are unsatisfactory because they do not mention Johnson. ■ Though (E) does ment\,ion Johnson, it is \,misleading in that \,the words “Johnson has” suggest that Johnson i\,s the owner rather than the painter of the portraits and landscapes\,. ■ (D) is correct because it properly identifies Johnson as the painter of the artworks and thus provides an antecedent for the pronoun “her” in sentence 11. Correct answer: (D) / Difficulty level: Easy The Essa\b The essay measures your ability to: ■ Develop a point of view on an issue presented in an excerpt; ■ Support your point of view using reasoning and examples from your reading, studies, experience, or observations; and ■ Follow the conventions of standard written English. Approaches to the Essa\b There are no shortcuts to success on the S\bT essay. You will not receive high scores on your essay just because it i\,s long, or has five paragraphs, or uses literary examples. The high school and college teachers who score the S\bT reward essays that insightfully develop a point of view with appropriate reasons and examples and that use\, language skillfully. So what can you do to write a successful S\bT essay? ■ Read the entire assignment\b It’s all there to help you\b Every essay assignment contains a short paragraph about the issue. Imagine that you are talking to the author of the\, paragraph about the issue. Would you argue with him or her\,, or agree? What other ideas o\,r examples would you bring up? \bnswering these questions wi\,ll help you develop your own point of view. ■ Don’t oversimplify\b Developing your point of view doesn’t mean coming up with as ma\,ny examples as you can. Rushing to give multiple relevant examples can lead you to oversimplify a complex topic. \bn essay with one or two thoughtful, well- developed reasons or examples is more likely to get a high score than an essay with three short, simplistic examples. ■ There’s nothing wrong with “I\b” You are asked to develop your point of view on the issue, not give a straight report of the facts. This is your opinion, so feel free to use “I,” and give examples that are meaningful to you, even ones from your personal life or experiences. Of course you need to support your ideas appropriately and show that you can use language well, but remember: The essay is an opportunity for you to say what you think about an \,issue relevant to your life. \birections The essay gi\bes you an opportunity to show how effecti\bely you can de\belop and express ideas. You should, therefore, take care to de\belop your point of \biew, present your ideas logically and clearly, and use language precisely. Your essay must be written on the lines pro\bided on your answer sheet — you will recei\be no other paper on which to write. You will ha\be enough space if you write on e\bery line, a\boid wide margins, and keep your handwriting to a reasonable size. Remember that people who are not familiar with your handwriting will read what you write. Try to write or print so that what you are writing is legible to those readers. Important Reminders: • A pencil is required for the essay. An essay written in ink will recei\be a score of zero. • \bo not write your essay in your test book. You will recei\be credit only for what you write on your answer sheet. • An off-topic essay will receive a score of zero. • If your essay does not reflect your original and individual work, your scores for the entire test may be canceled. • An electronic copy of your essay will be made available to each of your designated score recipients: colleges, universities, and scholarship programs. You ha\be twenty-fi\be minutes to write an essay on the topic assigned below. Think carefully about the is\,sue presented in the following excerpt and the assignment below. Many persons believe that to move up the ladder of \,success and achievement, they must forget the past, repress it, and relinquish it. But \,others have just the opposit\,e view. They see old memories as\, a chance to reckon with the past a\,nd integrate past and present. \bdapted from Sara Lawrence-Lightfoot, I’ve Known Rivers: Lives of Loss and Li\beration Assignment: Do memories hinder\, or help people in\, their effort to learn from the past and suc\,ceed in the present? Plan and write an essay in which you develop your point of view on this issue. Support your position with \,reasoning and examples taken from your reading, studies, experience, or observations.

30 Getting Ready for the SAT Sa\bple Essays Score of 6: Without our past, our future would be a tortuous path leading to nowhere. In order to mo\be up the ladder of success and achie\bement we must come to terms with our past and integrate it into our future. E\ben if in the past we made mistakes, this will only make wiser people out of us and guide us to where we are supposed to be. This past year, I was auditioning for the fall play, “Cat on a Hot Tin Roof.” To my detriment I thought it would be a good idea to watch the mo\bie in order to prepare. For two hours I studied Elizabeth Taylor’s mannerisms, attitude, and diction, hoping I could mimic her performance. I auditioned for the part of “Maggie” feeling perfectly confident in my portrayal of Elizabeth Taylor, howe\ber, I was unaware that my director saw exactly what I had been thinking. Unfortunately, I didn’t get the part, and my director told me that he needed to see “Maggie” from my perspecti\be, not Elizabeth Taylor’s. I learned from this experience, and promised myself I would not try to imitate another actress, in order to create my character. Perser\bering, I was anxious to audition for the winter play just two months later. The play was Neil Simon’s “Rumors,” and would get the opportunity to play “Chris,” a sarcastic yet witty role, which would be my final performance in high school. In order to de\belop my character, I planned out her life just as I thought it should be, ga\be her the \boice I thought was right, and the rest of her char acter unfolded beautifully from there. My director told me after the first show that “Rumors” was the best work he’d e\ber seen from me, and that he was amazed at how I’d de\beloped such a belie\bable character. Thinking back to my first audition I was grateful for that chance I had to learn and to grow, because without that mistake I might ha\be tried to base “Chris” off of someone I’d known or something I’d seen instead of becoming my own character. I utilized the memory of the Elizabeth Taylor debacle to impro\be my approach to acting and ga\be the best performance of my life so far. This essay effectively and insightfully develops its point of \,view (In order to move up the ladder of \gsuccess and achievement we must come to terms with our past a\gnd integrate it into our future) through a clearly appropriate extended example drawing on the writer’s experience as an actor. The essay exhibits outstandin\,g critical thinking by pre senting a well-organized and clearly focused narrative that aptly illus\,trates the value of memory. The essay also uses language skillfully, demonstrating meaningful variety in sentence structure (To my detriment I though\gt it would \be a good idea to watch the movie in order to prepare. For two hours I studied Eliza\beth Taylor’s mannerisms, attitud\ge, and diction, hopi\gng I could mimic her performance. I auditioned for the part of “Maggie” feeling perfectly confident in my portrayal of Eliza\beth Taylor, however, I was unaware that my director…). Despite minor errors, the essay demonstrates clear and consistent mastery and is scored a 6. Score of 5: I agree with Ms. Sara Lawrence-Lightfoot in saying that some people “see old memories as a chance to reckon with the past and integrate past and present.” Many people are so troubled by things that happened in their past that they are not able to focus on the present. For example, in the book Ceremony , by Leslie Marmon Silko, Tayo, the main character, can not concentrate on the present because he constantly hounds himself o\ber things that happened during World War II and his troubled childhood. Howe\ber, past memories can help people to succeed in the present. An historical example of people learning from the past would be the Marshall Plan. After the conclusion of World War II there were many countries around the world in need of economical assistence to help rebuild their war torn countries, and the United States would ha\be to be the one to pro\bide that assistence. Many American politicians thought it was foolish for the US go\bernment to spend money abroad on countries that would not be able to repay the loan for a long time. Howe\ber, George Marshall, a former general and later Secretary of State under President Truman, remembered how the exact same argument of “why should we spend money on war torn nations that really owe us reparations?” had been used after World War I towards Germany. The lack of assistence towards Germany after World War I had caused a gigantic economic depression in Germany that had made the Mark (German money) \birtually worthless. The German people became so desperate that they started supporting an extreme German nationalist named Adolf Hitler, who e\bentually started World War II. Marshall knew that if the US did not help war torn Germany and, especially, Japan, we could e\bentually ha\be a World War III on our hands. This focused essay effectively develops its point of view and demonstrates strong critical thinking (Many people are so trou\bled \by things that happened in their past that they are not a\ble to focus on the present. . . . However, past memories can help people to succeed in the present) . The essay uses appropriate reasoning and examples and demonstrates coherence and progression of ideas (Many American politicians thought it was foolish for the US government to spend money a\broad on countries that would not \be a\ble to repay the loan for a long time. However, George Marshall . . . remem\bered how the exact same argument . . . had \been used after World War I towards Germany) . The essay also exhibits facility in the use of language. To earn a score of 6, the writer needs to achieve clearer coherence and smoother progression of ideas by integrating the example of Ceremony more effectively into the overall essay, perhaps through an extended comparison of Tayo’s and Marshall’s experiences of World War II. The essay demonstrates reasonably consistent mastery and is scored a 5.

Getting Ready for the SAT 31 Score of 4: Interestingly enough, I fall in the middle of these statements. I belie\be that one should remember the past and learn from those e\bents. Howe\ber, I also belie\be that many bad memories harm the present and the future. The only way to continue, many times, is to forget and forgi\be. My brother, who is college, has pro\bed to me the importance of getting good grades and acti\bely participating in extracorrecular acti\bities. These two ideas helped him to get into the prestegious college of the Uni\bersity of Notre Dame. His education there will allow him to ha\be a prosperous career as an adult. Re\biewing these facts and ideas has led me to belie\be if I do the same, I will ha\be a similar promising career. Consequently, I ha\be gotten good grades and ha\be seen interest from many prestigious programs. Through my knowledge, I ha\be learned that in many bad instances, time to forget is \bery important. Ireland, for example, had been persecuted for many hundreds of years from 1000 AD to 1900 AD. After being granted the Irish Free State, they attacked many parts of Britain for retribution of those many years of being oppressed. Consequently there has been on going hostility between the two peoples. This hostility has cost the li\bes of many hundreds of people. A quote once said, “Violence begets \biolence” is the perfect phrase for this warfare. The only way to stop the loss of life is to forget and forgi\be; start anew. Different situations require different actions to proceed in a positi\be manner. Many times, people are required to use both elements. For example, let’s forget this part and concentrate on how to bring this positi\be part into light. Both of the ideas on remembering and forgetting ha\be their reasons for existing and both are positi\be. This essay provides adequate reasons and examples to support both aspects of its \,point of view (I \believe that one should remem\ber the past and\g learn from those events. However, I also \believe that many \bad memories harm \gthe present and the future), thus demonstrating competent critical thinki\,ng. The essay is generally organized and focused and features coherence and progression of ideas. Facility in the use of lan\,guage is adequate, despite some inconsistencies (Through my knowledge, I have learned that in m\gany \bad instances, time to forget is very important). The essay also has some err\,ors in grammar, usage, and mechanics. To earn a higher score, the writer should provide additional appropriate evidence and use critical \,thinking to extend the discussion of situations \,in which “people ar\,e required to use both elements\,.” The essay demonstrates adequate mastery and receives a 4. Score of 3: Memories can be helpful to some and hinder others. I belie\be that memories from different aspects of ones life ha\be different consequences. One memory may be bad and it may be best forgotten about, when trying to succeed. Though some memories may gi\be on strength to suceed in achie\bing a higher status in life. When a person completes a task they ha\be done once before, it trigers a memory and lets the reader reflect on that particular time in life. For example, a sporting team at the local high school makes it to the state championships, but se\berly loses to their opponent, the next time they get to the state championships they may think about the past and how they lost before, and it may hinder there feelings and they may once again lose. This demonstrates how a memory can ruin a certain acti\bity for e\ber. On the other hand a memory can also help someone to mo\be up the ladder of success. As an example if a person has cancer and is gi\ben treatment then diagnosed in remission they feel like they ha\be beat the cancer. When the patient in remission is later told that the cancer has grown back, the patient might feel that they can kill the cancer again because when looking at the past they see they ha\be beat it once why not beat it again. This demonstrates how a memory can be helpful to a person. In this case it did not help the person climb the ladder of success though it helped the to continue climbing the ladder of life to the extent that they were able to climb. Those two short examples just go to demonstrate how memories of the past can both help and hinder a person in their path of not only success but also in the path of life. This essay develops a point of vie\,w (Memories can \be helpful to some and hinder oth\gers) and shows some critical thin\,king by providing examples of the positi\,ve and negative effects of memories. However, the examples are limited in focus, featuring some lapses in coherence and progression of ideas, and a\,re thus inadequate to support the position. The essay also demonstrates occasional problems in sentence structure and mechanics. To achieve a higher score, this writer needs to use critical thin\,king to clarify and expand each example by adding additiona\,l focused reasoning and detai\,ls. The writer also needs to avoid using run-on sentences (. . . when looking a\gt the past they see they have \beat it once why not \beat it again). The essay demonstrates developing mastery and earns a 3.\,

32 Getting Ready for the SAT Score of 2: I think it is wrong to belie\be that to mo\be up the ladder of success and achie\bement, that they must forget the past, repress it, and relinquish it. E\berything you did and saw in the past helps you to mo\be on. E\bery single happy moment, e\bery mistake you make is getting a part of you. Your actions become habits which creates your personality and helps you to make your own experience. Therefore memories help people in their effort to learn from the past and succeed in the present. E\berything we do has to do with our experiences in the past, the way we get along with people or treat them, the way we turn out to be an adult. If you don’t li\be with making your own decisions, mistakes, and your experience with people and the world or school you won’t ha\be any examples to compare or to handle any coming situations in the future. If you get e\berything told you by someone, you will always wait for other people to make decisions for you and won’t ha\be your own point of \biew. For succeed you ha\be to know what you want, to find that out, you ha\be to ha\be been through some difficult situations in the PAST. \blthough it expresses a point of view (I think it is wrong to \believe that to move up the ladder of \gsuccess and achievement, that they must forget the past, repress it, and relinquish it), this essay is seriously limited, exhibiting weak critical thinki\,ng, insufficient use of evidence, and serious problems with progression of ideas. The essay also demonstrates frequent problems in usage, grammar, and sentence structure. To achieve a higher score, the writer needs to develop the point of vi\,ew with reasons and specific examples instead of merely repeating the same v\,ague ideas (Everything you did and saw in the past helps \gyou to move on. . . . Everything we do has to do with our experiences in the past). The essay demonstrates little mastery and is scored a 2. Score of 1: My oppion on this topic are oposing memories and fa\boring them. People do succed with repeating their memories. They might ha\be horrible memories but also succeed because they don’t repeat the past. I also think memories should not rule the present. If you let the past o\bercome the preset you won’t get any where. This is why memories should be guidelines, not rules. If you repeat the past it won’t come out as well as it did because the world has changed. See the past will ne\ber change with the world, but the world will change to o\bercome the past. So in conclusion don’t forget the past or li\be in it, and the past is only guidelines. This minimal essay demonstrates very little mastery, offering only a collection of general ideas in suppor\,t of the writer’s point of view (don’t forget the past or live in it, and the pa\gst is only guidelines). The evidence presented is disorganized and unfocused, resulting in a disj\,ointed essay. To earn a higher score, this writer needs to provide additional focused evidence that develops the point of \,view, including specific \,examples. The essay demonstrates very little mastery and receives a 1.

Getting Ready for the SAT 33 Official SAT Practice Test About the Practice Test Take the practice test, which starts on page 42, to reinforce your test-taking skills an\b to be more comfortable when you take the SAT. This practice test will give you a goo\b i\bea of what to expect on the actual test. Of course, the test you eventually take will \biffer in some ways. For example, its sections may be in a \bifferent or\ber. Your actual test will also be longer than the practice test, because it will contain an a\b\bitional 25-minute, unscore\b section (unless you are approve\b to take the SAT with exten\be\b time or a similar accommo\bation). The practice test originally containe\b an unscore\b section, Section 3, when it was given in a test center. Section 3 is omitte\b in this booklet, so you nee\b to set aside three hours and 35 minutes to take the test — 25 minutes less than the actual testing time you ’ll experience on test \bay. Approaches to the Practice Test The practice test will help you most if you take it un\ber con\bitions as close as possible to those of the actual test. ■ Plan to complete the entire test in one sitting. —You get three five-minute breaks. Take a break after every other 25-minute section, then work through all the shorter sections without a break. —Allow yourself the specifie\b amount of time for each section. Pace yourself by using a watch (without an au\bible alarm). ■ Sit at a \besk or table cleare\b of any other papers or books. Do not use any prohibite\b items such as \bictionaries, notes or scratch paper. ■ Use an acceptable calculator that is familiar to you for the mathematics section. ■ Rea\b the test instructions carefully. Marking the Answer Sheet Getting cre\bit for the right answer \bepen\bs on marking it correctly. On test \bay, an\b when filling out your answer sheet for the practice test, follow these important instructions: ■ Make sure you use a No. 2 pencil. ■ Fill in the entire circle on the answer sheet \barkly an\b completely. ■ If you change your response, erase it as completely as possible. Calculating Your Scores To score your test, follow the instructions on page 82. You’ll nee\b to count the right an\b wrong answers for each section, an\b then convert your “raw” score to the College Boar\b scale of 200 to 800. You will nee\b to choose a score for your essay. Use the Scoring Gui\be on page 81 to \betermine how your particular essay might be score\b. Reviewing Your Performance After you score your practice test, review your performance an\b ask yourself these questions: ■ Did I run out of time before I finished a section? Remember, all questions count the same. Be prepare\b to keep moving on test \bay an\b not spen\b too much time on any one question. ■ Did I make care\bess mistakes? You may have misrea\b the question, neglecte\b to notice a wor\b like “except” or “best,” or solve\b for the wrong value. Recognizing these small errors will help you avoi\b them on test \bay. ■ Did I spend too much time reading directions? Review any \birections that gave you trouble so you \bon’t have to spen\b as much time rea\bing them when you take the actual test. Visit collegeboard.org/sat\spracticetest to rea\b sample score\b essays an\b take an online practice test with automatic scoring. The Official SAT Online Course ™ ➢ Take a practice test online ➢ Receive an immediate essay score ➢ Practice with more tests and q\bizzes Visit: colle\beboard.or\b/satonlinecourse

34 Getting Ready for the SAT

Getting Ready for the SAT 35

36 Getting Ready for the SAT

Getting Ready for the SAT 37

38 Getting Ready for the SAT

Getting Ready for the SAT 39

40 Getting Ready for the SAT

Getting Ready for the SAT 41

42 Getting Ready for the SAT   ESSAY  Time —  25 minutes    Turn to page 2 of your answer sheet to write your ESSAY.  The essay gives you an opportunity to show how effectively you can develop and express ideas. \bou should, therefore, take   care to develop your point of view, present your ideas  logically and clearly, and use language precisely.     \bour essay must be written on the lines pr ovided on your answer sheet — you will r eceive no other paper on which to write.   \bou will have enough space if you write on  every line, avoid wide margins, and keep your handwriting to a reasonable size.  Remember that people who are not familiar with your handwriting  will read what you write. Try to write or print so that what  you are writing is legible to those readers.     Important Reminders\b     •  A pencil is required for the essay.  An essay written in ink will receive a score of zero.    •  Do not write your essay in your test book.  \bou will receive credit only for what you write on your   answer sheet.    •  An off-topic essay will recei ve a score of zero.     •  If your essay does not reflect your original and individual  work, your scores for the entire test may be canceled.     •  An electronic copy of your  essay will be made availabl e to each of your designated score recipients\b colleges,  universities, and scholarship programs.    \bou have twenty-five minutes to write an essay on the topic assigned below.        Think carefully about the issue presented in the following excerpt and the assignment below.     A colleague of the great scientist James Watson remarked that Watson was always “lounging  around, arguing about problems instead of doing experiments.” He concluded that “There is  more than one way of doing good science.” It  was Watson’s form of idleness, the scientist   went on to say, that allowed him to solve “the greatest of all biological problems:  the discovery  of the structure of DNA.” It is a point worth  remembering in a society overly concerned with  efficiency.   Adapted from John C. Polanyi, “Understanding Discovery”     Assignment\b Do people accomplish more when they are allowed to do things in their own way? Plan and write an essay   in which you develop your point of view on this issu e. Support your position with reasoning and examples   taken from your reading, studies, experience, or observations.          BEGIN WRITING \bOUR  ESSA\b ON PAGE 2 OF THE ANSWER SHEET.                  \bf you finish before time is called, you may check your work on this section only.  Do not turn to any other section in the test. 

Getting Ready for the SAT 43         SECTION 2  Time —  25 minutes  20 Questions    Turn to Section 2 (page 4) of your answer sheet to answer the \buestions in this section.    Dire\btions:   For this section, so lve each problem and  decide which is the best of the choi ces given. Fill in the corresponding  circle on the answer sheet. \bou may us e any available space for scratchwork.         1.  When  70, 000  is written as  7.0 10 , n ¥  what is the  value of  n ?  (A)  1  (B)  2  (C)  3  (D)  4  (E)  5          2.  On a car trip Sam drove  m  miles, Kara drove twice as  many miles as Sam, and Darin drove 20 fewer miles  than Kara. In terms of  m, how many miles did Darin  drive?  (A)  2 20 m +   (B)  2 20 m-   (C)  20 2 m +   (D)  20 2 m +   (E)  20 2 m -     3.  If x  and  y  are positive integers, what are all the  solutions  ( ), x y  of the equation  3 2 11?x y + =   (A)  ( )1, 4  only  (B)  ( )3,1  only  (C)  ( )1, 4  and  ( )2, 2   (D)  ( )1, 4  and  ( )3,1   (E)  ( )2, 2  and  ( )3,1    

44 Getting Ready for the SAT           4.  A company’s profit, P, in dollars, for producing  x  machines in one day is given \by  2 500 20 . P x x = -   If the company produces 10 m achines in one day, then,  according to this formula, wh at is the profit for that  day?  (A)  $5,000  (B)  $4,000  (C)  $3,000  (D)  $2,000  (E)  $1,000              12 , 12, 12n n - +     5.  What is the average (arithmetic mean) of the  3 quantities in the list a\bove?  (A)    4  (B)  12  (C)  18  (D)    4 3 n +   (E)  12 3 n +               6.  In isosceles triangle  ABC  a\bove,  AM  and  CM  are  the angle \bisectors of angle  BAC  and angle  BCA.  What is the measure of angle  AMC ?  (A)  110 °  (B)  115 °  (C)  120 °  (D)  125 °  (E)  130 °    7.  A fruit salad is made from pineapples, pears, and  peaches mixed in the ratio of  2 to 3 to 5, respectively,  \by weight. What fraction of the mixture \by weight is  pineapple?  (A)  1 5   (B)  3 10   (C)  2 5   (D)  1 2   (E)  2 3               8.  In the figure a\bove, square  RSTU  is inscri\bed in the  circle. What is the degree measure of arc  p ? ST   (A)    45∞   (B)    60∞   (C)    90 ∞  (D)  120 ∞  (E)  180 ∞            9.  If  P  and  Q  are two sets of nu m\bers, and if every  num\ber in  P  is also in  Q,  which of the following  CANNOT \be true?  (A)  4 is in \both  P  and  Q.  (B)  5 is in neither  P  nor  Q.  (C)  6 is in  P,  \but not in  Q.  (D)  7 is in  Q,  \but not in  P.  (E)  If 8 is not in  Q,  then 8 is not in  P.

Getting Ready for the SAT 45         10.  What is the maximum number of rectangular blocks  measuring 3 inches by 2 inches by \b inch that can be  packed into a cube-shaped box whose interior measures  6 inches on an edge?  (A)  24  (B)  28  (C)  30  (D)  36  (E)  40        11.  If  0 aπ  and  5 5 , a x x a+ = +  what is the value of  x ?  (A)  5 -  (B)  \b -  (C)  \b  (D)  2  (E)  5          12.  The figure above is composed of 25 small triangles that  are congruent and  equilateral. If the area of  DFH   is  \b0, what is the area of  ? A FK   (A)  40  (B)  42.5  (C)  50  (D)  52.5  (E)  62.5        3 2 2 \b9 3 \b4 x y z x y z + + = + + =   13.  If the equations above are true, which of the following  is the value of  ? y z+   (A)  5 -  (B)  4 -  (C)  0  (D)  4  (E)  5        14.  A boat costs  x  dollars, and this cost is to be shared  equally by a group of people. In terms of  x, how many  dollars less will each person contribute if there are  4 people in the group instead of  3 ?  (A)  \b2 x   (B)  4 x   (C)  3 x   (D)  7 \b2 x   (E)  7 x    15.  If  2 3 and 2, y x x= + <  which of the following  represents all the possible values for  ? y  (A)  7 y<   (B)  7 y>   (C)  5 y<   (D)  5 y>   (E)  5 7 y < <       16.  The graphs of the functions  f  and  g  in the interval  from  2 x= -  to  2x=  are shown above. Which of   the following could express  g  in terms of  f ?  (A)  ( ) ( ) \b g x f x = +   (B)  ( ) ( ) \b g x f x = +   (C)  ( ) ( ) \b \b g x f x = + +   (D)  ( ) ( ) \b g x f x = -   (E)  ( ) ( ) \b g x f x = -

46 Getting Ready for the SAT             17.  In the figure above, a shaded polygon which has e\bual  sides and e\bual angles is partially covered with a sheet  of blank paper. If  80,x y+ =  how many sides does  the polygon have?  (A)  Ten  (B)  Nine  (C)  Eight  (D)  Seven  (E)  Six                      18.  If  s,  t,  u,  and  v  are the coordinates of the indicated  points on the number lin e above, which of the  following is greatest?  (A)  s t +   (B)  s v +   (C)  s t -   (D)  s v -   (E)  s u +                       19.  On the day of a rainstorm, the depth of the water at a  certain location along the Winding River was recorded  hourly, and the results are  indicated in the line graph  above. Each unit on the vertical axis represents 1 foot.  If the depth of the water decreased 10 percent from  3:00  P.M. to 4:00  P.M., what was the depth of the water  at  4:00  P.M.?  (A)  3 feet  (B)  15 feet  (C)  18 feet  (D)  20 feet  (E)  30 feet        20.  For all numbers  a  and  , b let  a b  be defined by  . a b ab a b = + +   For all numbers  , x  ,y and  , z  which of the following must be true?  I.  x y y x =     II.  ( ) ( ) ( ) 1 1 1 x x x x- + = -    III.  ( ) ( ) ( ) x y z x y x z + = +      (A)  I only  (B)  II only  (C)  III only  (D)  I and II only  (E)  I, II, and III          S T O P   If you finish before time is called\b you may check your work on this section only.  Do not turn to any other section in the test. 

Getting Ready for the SAT 47       SECTION 4  Time —  25 minutes 24 Questions    Turn to Section 4 (page 5) of your answer sheet to answer the \buestions in this section.    Dire\btions:   For each question in this section, select  the best answer from among the choices given and  fill in the corres\bonding  circle on the answer sheet.    Each sentence below has on e or two blanks, each blank  indicating that something has been omitted. Beneath   the sentence are five words or sets of words labeled A  through E. Choose the word or set of words that, when  inserted in the sentence, best  fits the meaning of the  sentence as a whole.  Example:  Ho\bing to ------- the dis\bute, negotiators \bro\bosed   a com\bromise that they felt would be ------- to both   labor and management.  (A)  enforce . . useful  (B)  end . . divisive  (C)  overcome . . unattractive  (D)  extend . . satisfactory  (E)  resolve . . acce\btable                          1. Some fans feel that s\borts events are ------- only when  the com\betitors are of equal ability, making the  outcome of the game -------.  (A)  successful . . assured  (B)  boring . . questionable  (C)  dull . . foreseen  (D)  interesting . . \bredictable  (E)  exciting . . uncertain      2.  Alfred Schnittke’s musical com\bositions are -------:   \bhrases are cli\b\bed, broken into sections, and s\blit  a\bart by long rests.  (A)  garnished    (B)  im\brovisational    (C)  fragmented    (D)  cautious    (E)  uniform      3.  The consumer advocate cl aimed that while drug  manufacturers ------- the su\b\bosed advantages   of their \bro\brietary brands, generic versions of   the same medications are often equally -------.  (A)  tout . . efficacious  (B)  research . . innocuous  (C)  market . . \brohibitive  (D)  laud . . counter\broductive  (E)  extract . . \brescri\btive      4.  Latoya’s ------- is shown by her ability to be -------:   she can see her own faults  more clearly than anyone  else can.  (A)  \berce\btiveness . . self-centered  (B)  objectivity . . restrictive  (C)  cynicism . . self-destructive  (D)  o\ben-mindedness . . com\blacent  (E)  insightfulness . . self-critical      5.  The bearded dragon lizard is a voracious eater, so -------  that it will consume as many insects as \bossible.  (A)  abstemious    (B)  cannibalistic    (C)  slovenly    (D)  insatiable    (E)  un\balatable      6.  Because drummer Tony W illiams \baved the way for  later jazz-fusion musi cians, he is considered a -------   of that style.  (A)  connoisseur    (B)  revivalist    (C)  beneficiary    (D)  dis\barager    (E)  \brogenitor      7.  The \bolitician’s s\beech to the crowd was com\bosed of  nothing but -------, a bitter railing against the \barty’s  o\b\bonents.  (A)  digressions    (B)  diatribes    (C)  \blatitudes    (D)  machinations    (E)  acclamations      8.  Favoring economy of ex\bression in writing, the  \brofessor urged students toward a ------- rather   than an ------- \brose style.  (A)  s\bare . . ornate  (B)  terse . . o\binionated  (C)  \bersonal . . academic  (D)  baroque . . embellished  (E)  re\betitive . . intricate   

48 Getting Ready for the SAT       The passages below are followed by questions based on their cont ent; questions following a pair of related passages \bay also   be based on the relationship between the paired passages. Answer the questions on the basis of what is stated  or i\bplied  in the  passages and in any introductory \baterial that \bay be provided.    Questions 9-12 are based on the following passages\b  Passage 1  Food has always been considered one of the \bost salient  \barkers of cultural traditions . When I was a s\ball child,  food was the only thing that helped identify \by fa\bily as  Filipino A\berican. We ate  pansit lug-lug (a noodle dish)  and \by father put  patis (salty fish sauce) on everything.  5  However, even this connection lessened as I grew older.   As \by parents beca\be \bore acculturated, we ate less  typically Filipino food. When I was twelve, \by \bother  took cooking classes and l earned to \bake French and  Italian dishes. When I was in high school, we ate chicken  10  \barsala and shri\bp fra diablo \bore often than Filipino  dishes like  pansit lug-lug .  Passage 2  Jean Anthel\be Brillat-Savarin — who in 1825 confi-  dently announced, “Tell \be what you eat, and I will tell  you who you are” — would have no trouble describing  15  cultural identities of the United States. Our food reveals   us as tolerant adventurers who do not feel constrained   by tradition. We “play with our food” far \bore readily   than we preserve th e culinary rules of ou r varied ancestors.  A\bericans have no single national cuisine. What unites  20  A\berican eaters culturally is how we eat, not what we   eat. As eaters, A\bericans \bing le the culinary traditions   of \bany regions and cultures. We are \bultiethnic eaters .     9\b  Which of the following state\bents best captures   the relationship between the two passages?  (A)  Passage 1 notes proble\bs for which Passage 2  proposes solutions.  (B)  Passage 1 presents clai\bs that are debunked   by Passage 2.  (C)  Passage 2 furnishes a larger context for   the experiences described in Passage 1.  (D)  Passage 2 provides an update of the situation  depicted in Passage 1.  (E)  Passage 2 uses \baterial presented in Passage 1   to correct a popular \bisconception.    10\b  The author of Passage 2 would \bost likely regard   the \bother’s willingness to “\bake French and   Italian dishes” (lines 9-10, Passage 1) as  (A)  laughably pretentious  (B)  understandably conservative  (C)  typically A\berican  (D)  a regrettable co\bpro\bise  (E)  a surprising attitude    11\b  The two passages differ in their discussions of food  pri\barily in that Passage 1  (A)  considers specific dish es eaten by particular  people, whereas Passage 2 co\b\bents on   a culture’s general attitude toward eating  (B)  contrasts the cuisines of different cultures,  whereas Passage 2 e\bphasizes culinary  practices co\b\bon to all cultures  (C)  presents an abstract  theory of food, whereas  Passage 2 offers a historical analysis of  consu\bption  (D)  e\bphasizes the role of nostalgia in food  preferences, whereas Passage 2 rejects   that approach as overly senti\bental  (E)  outlines so\be popular choices in cuisine,   whereas Passage 2 underscores those   that are \bore unusual    12\b  Unlike the author of Passage 2, the author of   Passage 1 \bakes significant use of  (A)  direct quotation  (B)  sociological analysis  (C)  hypothetical assu\bptions  (D)  historical sources  (E)  personal experience    Line

. . . . . .